Sunteți pe pagina 1din 68

DENTAL

QUEST

AIPG -2011

AIPG-2011
GENERAL ANATOMY
1) Only sensory branch of anterior division of mandibular nerve is a) Auriculotemopral nerve b) Lingual c) Long buccal d) Buccal nerve ANS- (c) Long buccal Nerve (B D Chaurasias Human Anatomy,3rd ed Vol II page no 123) Repeat AIPG 2009 Guys to be specific about buccal branch of anterior division of the mandibular nerve, It should be called as long buccal nerve( buccinator nerve) which is sensory to the buccal cortical plate, buccal mucosa and buccal gingiva. And remember the Buccal nerve is actually the terminal branch of facial nerve which supplies the Buccinator muscle. So please dont get confused. Remember the word LONG 2) Anterior ethmoidal nerve supplies all except a) Maxillary sinus b) Interior of nasal cavity c) Dural sheath of anterior cranial fossa d) Ethmoidal air cells ANS-(a) Maxillary Sinus (B D Chaurasias Human Anatomy III pg 234)

A I P G 2 0 1 1

Ant.Ethmoidal nerve is the branch of nasociliary nerve (a branch of ophthalmic div. of trigeminal nerve). Anterior ethmoidal nerve divides into internal and external nasal branches. Internal branch supplies the anterior superior parts of the nasal septum and wall. Before descending along the sides of crista galli into the nose it supplies anterior ethmoidal cells and dura of the anterior cranial fossa Nerve supply for maxillary sinus is from the infraorbital and superior alveolar (posterior, middle and anterior) branches of the maxillary nerve. These alveolar nerves run down to the teeth in the walls of the sinus.

3) a) b) c) d)

The sensory supply of the palate is through all of the following, except: Facial nerve Vagus nerve(hypoglossal according to some students) Glossopharyngeal nerve Trigeminal nerve ANS-(b) Vagus N or Hypoglossal N (B D Chaurasias Human Anatomy 4th ed Vol III pg no.212) General sensory nerves of the palate:

Middle and Posterior (lesser) Palatine nerves, which are branches of Maxillary nerve through the Pterygopalatine ganglion And from the glossopharyngeal nerve

Special sensory (Gustatory) nerves

DENTAL QUEST Your Quest For The Success In Post Graduate Dental Entrance !

DENTAL

QUEST

AIPG -2011

Carrying taste sensations from the oral surface are contained in the Lesser Palatine nerves. The fibres travel through the Greater Petrosal nerve to the geniculate ganglion of the Facial nerve and from there to the nucleus of the solitary tract. Also Remember Motor supply All the muscles of the soft palate is supplied by the cranial accessory nerve through vagal branches except for tensor palati which is supplied by the mandibular nerve. Guys,we also found in some text books that pharyngeal plexus is formed by both vagus and glossopharyngeal but even then VAGUS supplies motor fibers and not sensory. 4) Cranial nerve not carrying parasympathetic fibres a) Fourth b) Seventh c) Third d) Ninth ANS-(a) Fourth (Snells Anatomy 6th ed page no 706-707) Now why Option (a) is right, because Sympathetic nervous system has a thoracolumbar outflow i.e., the neurons leave spinal cord via ventral rami of T1 L4. Where as parasympathetic system has a craniosacral outflow. Its neurons travel in the cranial nerves (III,VII,IX, X) and in ventral rami of S1 S4 . 5) Premature synostosis of coronal sutures along with basal sutures shows which of the following features a) Brachycephaly b) Oxycephaly c) Trigonocephaly d) Plagiocephaly Ans-(b) Oxycephaly (O.P Ghai 17th ed .)Repeat AIPG 2007 Oxycephaly

A I P G 2 0 1 1

Oxycephaly, also known as Turricephaly and High-head syndrome Most severe form of craniosynostosis. It has premature closure of coronal suture with any other suture or can be premature fusion of all sutures Also known as acrocephaly or turricephaly Brachycephaly Premature closure pf coronal suture Trigonocephaly Premature fusion of metopic suture Dolicocephaly or scaphocephaly Premature closure of saggital suture.

Its relatively harmless anamoly.

6) a) b) c) d)

Geniculate ganglion is associated with Lacrimation Taste sensation Salivation Sweating

DENTAL QUEST Your Quest For The Success In Post Graduate Dental Entrance !

DENTAL

QUEST

AIPG -2011

Ans (b)Taste sensation (B.D.Chaurasia Human anatomy 4th ed page no 141)

Pterygopalatine ganglion ( sphenopalatine) Lacrimation and Salivation. Submandibular ganglion - salivation Otic ganglion salivation Geniculate ganglion Taste sensation

7)

Which of the following does not have not a somatic efferent component. a) Trochlear b) Facial c) Abducens d) Occulomotor ANS-(b) Facial N (Grays Anatomy Page no 800-801 2nd edition)Repeat AIPG 2008 Somatic efferent components are those which supply muscles. Facial nerve also supplies muscles. But the facial muscles are derived from the Branchial arches and hence are supplied by special efferent component Facial nerve does not have a somatic efferent component

A I P G 2 0 1 1

Functional Components of the Facial Nerve 1. Special visceral efferent (SVE) or Branchial: to muscles responsible for facial expression and for elevation of the hyoid bone. 2. General visceral efferent (GVE) or Parasympathetic: are secretomotor to the submandibular and sublingual salivary glands, the lacrimal gland, and glands of the nose, the palate, and the pharynx. 3. General visceral afferent (GVA): carries afferent impulses from the above mentioned glands. 4. Special visceral afferent (SVA): carry taste sensations from the anterior two-third of the tongue except from vallate papillae and from the palate. 5. General somatic afferent (GSA): probably innervate a part of the skin of the ear. 8) Parotid fascia continues anteriorly as? a) Fascia lata b) Deep cervical fascia c) Massetric fascia d) Carotid fascia ANS-(c) Massetric Facia (Grays anatomy,39 th edition pg 499,topic parotid fascia ) The superficial the layer of deep cervical fascia splits to enclose the parotid gland. Over the lateral surface of gland it is called as parotid fascia. Parotid fascia The parotid fascia sends numerous septae passing among the lobules of glandular tissue. Superiorly it is fused to the zygoma. Inferiorly it is continous as deep cervical fascia. Posteriorly fascia is fused to acoustic meatus and anterior border of sternocleido mastoid. Anterior to the gland, the fascia continues to invest the masseter. The deep fascia of the medial side is thinner and continous with the fascia of styloid muscles forms a part of stylomandibular ligament before fusing with the fascia of Digastric muscle and periosteum at the angle of the mansible. 9) a) Not attached to oblique line of thyroid cartilage Superior constrictor

DENTAL QUEST Your Quest For The Success In Post Graduate Dental Entrance !

DENTAL

QUEST

AIPG -2011

b) Inferior constrictor c) Sternothyroid d) Thyrohoid ANS-(a) Superior Constrictor (B D Chaurasias Human Anatomy 3h ed Vol III page no. 221)

o o
muscle.

Oblique line represents the point of attachment of three muscles: sternothyroid, thyrohyoid The Sternothyroid Muscle-- Superior attachment: oblique line of thyroid cartilage. The Thyrohyoid Muscle-- Inferior attachment: oblique line of thyroid cartilage Inferior constrictor has two rings Thyropharygeus arising from the oblique line of the thyroid and Cricopharyngeus arising from the cricoids cartilage behind the origin of cricothryroid

A I P G 2 0 1 1

10) Coeliac plexus is located: a) Anterolateral & around the aorta b) Posterolateral & around the aorta c) Anteromedial to lumbar sympathetic chain d) Posterolateral to lumbar sympathetic chain ANS-(a) Anterolateral to the aorta (B D Chaurasias Human Anatomy 4th ed Vol II Pg No 321 ,320 ; Grays Anatomy 40 Ed ,Page No 1045 , 1043) Coeliac plexus Also knows as Solar plexus is the largest major autonomic plexus. It is basically a dense network uniting two large coeliac ganglia which lie on each side of coeliac trunk. Celiac plexus is located anterolateral to the aorta near the origin of celiac artery. It supplies visceral sensory fibers to the upper abdominal viscera including pancreas, liver, stomach, gall bladder, spleen, proximal bowel upto transverse colon. It surrounds the coeliac trunk and the root of the superior mesenteric artery. It lies anterior to the crura of the diaphragm and the beginning of the abdominal aorta at the level of T 12 & LI . The lumbar sympathetic chain is located at the anterolateral border of the lumbar vertebral bodies (not the antero medial or posteriomedial as given in options C and D). An effective means of managing pain arising from the upper abdominal viscera is Coeliac plexus neurolysis.

BIOCHEMISTRY
11) Decreased BMR is seen in a) Obesity b) Hyperthyroidism c) Feeding d) Exercise ANS- (a) Obesity (Ganong's Review of Medical Physiology, 23nd ed page no 462)

Basal metabolic rate (BMR), and the closely related resting metabolic rate (RMR), is the amount of daily energy expended while at rest in a neutrally temperate environment, in the postabsorptive state (meaning that the digestive system is inactive, which requires about twelve hours of fasting in humans). Higher the muscle portion greater is the BMR. The BMR decreases as fat content proportionately increases.

DENTAL QUEST Your Quest For The Success In Post Graduate Dental Entrance !

DENTAL

QUEST

AIPG -2011

BMR decreases with age. BMR is less in females due to high fat content compared to males. BMR is increased in Hyperthyroidism, fever, exercise, cushing syndrome, tumors of adrenal gland. BMR is decreased in hypothyroidism, starvation, malnutrition, hypopituitarism, hypoadrenalism (eg, Addison's disease), and anorexia nervosa. Obesity may be associated with decreased BMR and is the single best answer amongst the option provided.

12)

Rothras test is used for the detection of a) Ketones b) Glucose c) Fatty acid d) Proteins ANS-(a) Ketone Bodies (Text book of Biochemistry by SATYANARAYAN 3rd ed Pg No 295) Rothras test for detection of ketone bodies in urine

A I P G 2 0 1 1

13)

In Niemann Picks disease the following substance accumulation in CNS is in excess? a) Glycerophosphatidase b) Phosphoinositides c) Sphingomyelins d) Glucocerebrosides ANS-(c) Sphingomyelin (Text book of Biochemistry by SATYANARAYAN 3rd ed Pg No 307)

Niemann pick disease is due to defect in enzyme sphingomyelinase which result in accumulation of sphingomyelins in liver,spleen and CNS There is a deficiency of the enzyme known as acid sphingomyelinase (ASM), which is required for breakdown of sphingomyelin. Deficiency of the enzyme is the cause of Niemann-Pick disease (sphingomyelin lipidosis) in which there is an accumulation of Sphingomyelin. 14) Type of collagen present in hyaline cartilage a) Type-I b) Type-II c) Type-III d) Type-IV ANS (b) Type II (HARPER Illustrated Biochemistry 26TH ED/PG 551,552) Collagen tissue specific, i.e. types II, IX, X and XI are found in hyaline cartilage Type II is the most abundant and common in cartilage.

PHYSIOLOGY
15) a. b. Hypercapnia results due to Low blood ph Low H+ ion concentration c. Decrease in co2 tension d. Hyperventilation ANS-(a) Low blood pH (Ganong's Review of Medical Physiology, 22nd ed page no 692)

DENTAL QUEST Your Quest For The Success In Post Graduate Dental Entrance !

DENTAL

QUEST

AIPG -2011

Hypoventilation leads to increased CO2 tension and H+ ion concentration in arterial blood and tissues. Thus there is increased acidity in blood or decreased pH. To offset the acidity compensatory mechanisms takes place. This is characterized by hyperventilation and washes out of CO2 accumulated. 16) Bronchial circulation is associated with a. Air conditioning b. Drug absorption c. Gaseous exchange d. Reserve volume ANS (a) Air Conditioning (Reff Pulmonry Physiology by Michael G Levitzky 6th ed - Page 87) The Respiratory System This system has 3 functions: Air conduction; Air filtration; Gas exchange (respiration). Air passages consist of Conducting portion and Respiratory portion. The conducting portion is the air passages that lead to the sites of respiration so gas exchange can occur. Capillaries within the lungs come into intimate contact with the alveoli and are the structural basis of gas exchange in the lung parenchyma. Conditioning of the air before it reaches the respiratory portion occurs and consists of warming, moistening, and removal of particulate material. A mucous and serous secretion is very important in the conditioning process and also stops dehydration of the underlying epithelium 17) Differential leukocytes count is used for diagnosis of a. Anemia b. Leukocytes c. Leukopenia d. Eosinophilia Ans (d) Eosinophilia (Textbook of Practical Physiology By G.K. Pal, 1st ed page no 100 and Nelima malik 1st ed page 13 and National Boards)

A I P G 2 0 1 1

There is a direct method and indirect method for estimation of number Eosinophilia.The indirect method is more preferred and used Differential leukocyte count. Percentage of normal eosinophil count in DLC is 24% 18) a. b. c. Christmas disease occurs due to deficiency of Factor IX Factor X Factor XI d. Factor.VII ANS-(a) FACTOR IX (Ganong's Review of Medical Physiology, 20th ed page no 522)

FACTOR IX is Christmas factor and its absence causes Christmas disease Factor VIII deficiency is called as classical hemophilia.

DENTAL QUEST Your Quest For The Success In Post Graduate Dental Entrance !

DENTAL

QUEST

AIPG -2011

19) Which of the following hypothalamic nucleus is mainly involved in the secretion of ADH a. Pre optic b. Supra optic c. Para ventricular d. Suprachiasmatic ANS (b) Supraoptic nucleus (Human physiology by Chowdary 2nd ed , pg no 446) The human pituitary gland consists essentially of two parts both controlled by the hypothalamus.

A I P G 2 0 1 1

The glandular part is the adenohypophysis or anterior lobe, and the neural part is the neurohypophysis or posterior lobe. Anterior lobe of pituitary Adenohypophysis Acidophilic cells produce (Somatotropin) Prolactin

Growth

hormone

Basophilic cells produce TSH,(Thyroid stimulating hormone)ACTH, FSH (Gametokinetic factor) LH. Middle lobe of pituitary Posterior pituitary Secretes melanocyte stimulating hormone (MSH) Neuro hypohysis Secretes ADH and Oxytocin

Both the Supraoptic and Paraventricular nuclei exert control over both the harmones ADH and Oxytocin. But in specific ADH is secreted mainly by Supraoptic nucleus of hypothalamus and oxytocin mainly by paraventricular nucleus. From hypothalamus these hormones are transported to posterior pituitary through hypothalamo-hypophyseal tract by axonal flow. 20) a. b. c. Which of the following is true of substance P It mainly mediates pain sensation at primary neurons It has inhibitory effect in the descending pathways that modulate pain Substance p is a neurotransmitter that acts as a regulator of pain in brain

DENTAL QUEST Your Quest For The Success In Post Graduate Dental Entrance !

DENTAL

QUEST

AIPG -2011

d. All of the above.3 Ans (a) It mainly mediates pain sensation at primary neurons. (Mainly Mediates Pain sensation at Primary Neurons and Ganong's Review of Medical Physiology, 22nd ed page no 11) Substance P

Mainly released by the peripheral sensory free endings. Excitory in nature Only registers pain in the brain but cannot regulate it.

A I P G 2 0 1 1

It is a neuropeptide: an undecapeptide that functions as a neurotransmitter and as a neuromodulator It belongs to the tachykinin neuropeptide family.

Guys We also found this line in Ganong Substance P is released from the terminals of specific sensory nerves, it is found in the brain and spinal cord, and is associated with inflammatory processes and pain. Substance P functions as a neurotransmitter at the central endings (in the spinal marrow) to pass nociceptive information from the primary to the secondary neurons

Neurotransmitters are endogenous chemicals which transmit signals from a neuron to a target cell across a synapse *Neurotransmitter- one presynaptic neuron directly influences a postsynaptic partner (one neuron reaching one other neuron), **Neuromodulation (one neuro uses different neurotransmitters to connect to several neurons ). .Neuromodulatory transmitters secreted by a small group of neurons diffuse through large areas of the nervous system, having an effect on multiple neurons.

Examples of neuromodulators include dopamine, serotonin, acetylcholine, histamine and others. Nociception is defined as the neurochemical process by which pain signals are transmitted from the periphery to the central nervous system ( ascending) and back to periphery( descending). GABA in central nervous system and Serotonin acts as a regulator of pain perception through its effect on afferent neurons thereby decreasing intensity of pain signals to brain. It is also suggested that serotonin acts in the thalamus as a regulator of pain

21)

If bile acid pool is small,recycling is a. Fast b. Slow c. Normal rate d. There will be no recycling ANS (a) Recycling is Fast (Hepatology: a textbook of liver disease: Volume 1 by David Zakim-3/387)

This movement of molecules from the biliary tract to the small intestine and back to the liver followed by resecretion into bile is termed the enterohepatic circulation

There is an inverse relationship between the size and recycling frequency of the bile acid pool, so that secretion rate and hepatic return of bile acids remain constant,

DENTAL QUEST Your Quest For The Success In Post Graduate Dental Entrance !

DENTAL

QUEST

AIPG -2011

despite a wide range of pool sizes. In other words, the smaller the pool size, the faster the recycling frequency. At the hepatocyte level, a decreased return of bile acids to the hepatocyte is followed by increased bile acid biosynthesis; the signal appears to be the intracellular concentration of bile acids. Normally, bile acid synthesis is down regulated. 22) Which of the following controls voluntary eye movements a. Frontal eye field b. Superior colliculus c. Visual cortex d. Cerebellum Ans (a) Frontal Eye field (Physiology by Guyton 11th/617,690) Types of eye movements

A I P G 2 0 1 1

o o o o

VoluntaryControlled by frontal eye field Eg: .Saccade Frontal eye field controls voluntary changes of the direction of the gaze Involuntary --Controlled by occipital cortex. Eg: Vergence - convergence and divergence

DENTAL QUEST Your Quest For The Success In Post Graduate Dental Entrance !

DENTAL

QUEST

AIPG -2011

10

Frontal eye fields (FEF) The frontal eye fields (FEF) is a region located in the premotor cortex, which is part of the frontal cortex of the primate brain It plays a role in purely sensory processing and that it belongs to a fast brain system through a superior colliculus medial dorsal nucleus FEF ascending pathway.

23)
a. b. c. d.

Somatomedin mediates Deposition of Chondritin sulfate Lipolysis Gluconeogenesis Decreased rate of glucose uptake BY cells

A I P G 2 0 1 1

ANS (a) Deposition of Chondritin sulfate (Ganong's Review of Medical Physiology, 23nd ed page no 382) Somatotropin Somatomedin Another name for growth hormone secreted by somatotropes of anterior pituitary

Somatostatin

GH stimulates the liver and skeletal tissues to secrete somatomedin. Somatomedin is active form of growth harmone. GH does not have any direct action on bones. It acts through somatomedin. Somatostatin is an inhibitory hormone of growth hormone It is secreted by delta cells of islets of langerhans in pancreas.

Most of the anabolic actions of growth hormone are an indirect result of increased production of growth factors, which are called somatomedins, or insulin-like growth factors (IGFs). A major growth factor is somatomedin C, also called IGF-I.

Lorain dwarfism is due to deficiency of somatomedin while pituitary dwarfism or dwarfism is due to deficiency of growth harmone in children

Previously somatomedin was originally termed "sulfation factor" by its discoverers, Salmon and Daughaday, because it induced incorporation of sulfate into cartilage as chondroitin sulfate. In the ensuing years it has become known as "somatomedin.

Most bioassays of somatomedin are based on the incorporation of sulfate into cartilage. The incorporation of labeled sulfate into cartilage as chondroitin sulfate provides a useful index of cartilage metabolism 24) Which of following is not present in basal lamina a. Rhodopsin b. Entactin c. Laminin d. Integrin ANS (a) Rhodopsin (Oral development and histology by James Avery 3rd edition page 294 ) Basal lamina is present underlying the epithelium

It is made up of type IV collagen, glycoproteins such as laminin, nidogen/entactin, and proteoglycans. It is also composed of Integrins which are a family of transmembrane linker proteins. SO guys... Rodopsin is surely absent

DENTAL QUEST Your Quest For The Success In Post Graduate Dental Entrance !

DENTAL

QUEST

AIPG -2011

11

25)
a. b. c. d.

Pulmonary endothelium is not concerned with which of the following Lipoprotein lipase Plasma activation factor Thrombin Factor x

Ans-(d) Factor X (The Pulmonary Endothelium: Function in Health and DiseaseBy Norbert Voelkel, 1st ed page no 104-110)

A I P G 2 0 1 1

Lipoprotein lipase is a surface active Protein pulmonary endothelium has catalytic properties Pulmonary endothelium also possesses heparin like glycosaminoglycans and sulfated proteoglycans on their surface that sequester AT III and thrombin from the circulation, facilitating their reaction. Among Given options Factor X (or staurt factor-auto prothrombin) is not related to pulmonary endthelium 26) CO poisoning causes a. Hypoxic hypoxia b. Oxygen dissociation curve shifts to left c. Cyanosis d. Diffusion capacity of lungs decreases ANS (c) Cyanosis (Ganong's Review of Medical Physiology, 22nd ed page no 690,684,661) In carbon monoxide poisoning, oxygen cannot be transported and released to body tissues thus resulting in hypoxia.This is histotoxic anemia SO guys.. Now we are left with Cyanosis.. which is actually obvious due to absence of oxygen Many factors influence the affinity of this binding and alter the shape of the curve: LEFT SHIFT (HIGH AFFINITY FOR O2) low low low high high (alkalosis) foetal haemoglobin RIGHT SHIFT (LOW AFFINITY FOR O2) High High High Low low (acidosis) adult haemoglobin

TEMPERATURE 2.3-DPG p(CO2) p(CO) pH (BOHR EFFECT) TYPE OF Hb

27) Which of the following does not have a corresponding releasing hormone from a. FSH b. TSH c. ACTH d. Prolactin ANS- (d) Prolactin (Ganong's Review of Medical Physiology, 21st ed page no 251) The Tropic harmone releasing factors produced in the hypothalmus are transported into the anterior pituatory through Hypothalamus hypophyseal portal tract

DENTAL QUEST Your Quest For The Success In Post Graduate Dental Entrance !

DENTAL

QUEST

AIPG -2011

12

The hypothalamus plays important stimulatory role in regulating the secrection of ACTH ,beta LPH,TSH ,GH ,FSH and LH It also regulates prolactin secretion but its effect is predominantly inhibitory rather than stimulation 28) a. b. c. d. An important non respiratory function of lung is Anion balance Sodium balance Pottasium balance Calcium balance

A I P G 2 0 1 1

ANS (b)Sodium Balance (Principles of physiology by Paramanik 1st ed pg no 134)

Lungs also Converts angiotensin I to angiotensin II by the action of angiotensinconverting enzyme( this helps in sodium balance) Alter the pH of blood by facilitating alterations in the partial pressure of carbon dioxide Filter out small blood clots formed in veins Immunoglobulin-A is secreted in the bronchial secretion and protects against respiratory infections.

29) Main function of serum albumin


a. b. c. d. Oncotic pressure Toxin transport Buffer Drug

transport

ANS (a) Oncotic pressure (Textbook of Medical Biochemistry by Dinesh Puri 3rd ed page no 87) Guys... Yes. infact all are the functions of albumin. But why answer is Oncotic Pressure. Around 80% of plasma oncotic pressure is controlled by albumin Around 15% of buffering capacity is by albumin Now we dont have to look for other options as they are not the major functions, from above lines which says 80% surely is the maxm albumin can do so a is the right answer

GENERAL PATHOLOGY
30)
Occupational cancer are common in a. Liver b. Lung c. Skin d. Leukemia Ans (b) Lung (http://www.WHO.int/occupational_health/publications/newsletter/gohnet11e.pdf and Encyclopaedia of occupational health and safety, Volume 1; Volume 5 By Jeanne Mager Stellman 2.17) Its a WHO website saying friends and we definitely rely on it.

Most commonly affected is lung and least effected is liver here seems to be different views by different authors on the prevalence of occupational

cancers

DENTAL QUEST Your Quest For The Success In Post Graduate Dental Entrance !

DENTAL 31)

QUEST

AIPG -2011

13

Mesothelium of the lung is the most common involved structure.

Which of the following are least form of occupational cancers (another version of the above question) a. Lungs b. Liver c. Skin d. Leukemia Ans (b) Liver Cancer Among the given options liver cancer is the least occupational cancer. 32) a. b. c. d. Earliest transient changes following tissue injury Neutropenia Neutrophilia Monocytosis Lymphocyosis

A I P G 2 0 1 1

ANS-(b) Neutrophilia Frequently repeated question (Robbins & Cotran Pathologic Basis of Diseases 7th ed pg no 56) Neutrophilia predominates in the inflammatory infiltrate during the initial 24 hrs after tissue injury.

33)

Bence Jones proteins are derived from a. Alpha Globulin b. Beta Globulin c. Gamma Globulin d. Delta Globulin ANS. (c ) Gamma globulin . (Harrisons Internal Medicine 16th ed page no 657 , Primer on kidney diseases 4th ed page no 256; and Clinical laboratory medicine 6th /350) Seen in multiple Myeloma

Bence jones proteins are composed of light chains of type found in normal or pathological gamma globulin .

34)
a. b. c. d.

All are true about Bordetella pertusis except Infection with pertussis induces temporary natural immunity At present no antitoxins are available Cause disease in adults who have taken vaccine in child hood The acellular vaccine is associated with high incidence of neurological symptoms.

Ans (d) Acellular vaccine is associated with high incidence of neurological symptoms (Textbook of Microbiology By Ananthanarayan and Paniker7th ed page no 344) Cellular vaccine causes high incidence of neurological symptoms and not the acellular vaccine Pertussis, also known as whooping cough ( 100 days cough) Caused by Bordetella pertussis Immunization against pertussis does not confer life-long immunity. The duration of protection is between five to ten years. The newer acellular vaccine, known as DTaP, has greatly reduced the incidence of adverse effects compared to the earlier "whole-cell" pertussis vaccine with indication from 10-64 years of age. There is no known antitoxin. Treatment with effective antibiotic (erythromycin or azithromycin) shortens the infectious period but does not generally alter the outcome of the infection.

DENTAL QUEST Your Quest For The Success In Post Graduate Dental Entrance !

DENTAL

QUEST

AIPG -2011

14

35)

All of the following can be a cause of resurgent malaria except ? Genetic alterations in the host b. Drug resistance in parasite c. Resistance of the vector to DDT d. Antigenic variation in parasites ANS-(a) Genetic alterations in the host (Emergency infection by Scheld, Craig , Hughes vol21st ed page no 205)

a.

Resurgence of malaria includes drug resistance in the parasite that causes the disease, as well as resistance of the vector mosquito to insecticides, environmental factors and counterfeit medicines. Genetic alterations in the host has no role in the resurgence of malaria Plasmodium vivax followed by falciparum are common in resurgent malaria 36) a. b. c. d. Down syndrome translocation occurs as which of the following 46XY -14 t[14,21] 47XY +21 t[14,21] 47XY ,14 t[14,21] 46XY t{14,21]

A I P G 2 0 1 1

ANS-(d) 46XY, 14 t [14,21] (PreTest biochemistry and genetics By Cheryl Ingram-Smith, Kerry S. Smith. 2nd ed Page no 308) 37) Prostate specific antigen is used as a. Tumor marker b. Proto oncogene c. Oncogene d. Bacterial antigen ANS-(a) Tumor Marker (Robbins & Cotran Pathologic Basis of Diseases 7th page no 668-689) Repeat AIPG 2010 Prostate-Specific Antigen The PSA screening test is a blood test that looks for a specific tumor marker. The most commonly tested tumor marker for the prostate gland is prostate specific antigen. It is normally present in low levels in the blood of all adult men. The normal range is 0 to 4 ng/ml.

PSA is prostate-specific, not cancer-specific. A variety of conditions can raise PSA levels: prostatitis (prostate inflammation), benign prostatic hypertrophy (prostate enlargement), and prostate cancer. PSA levels can also be influenced by a number of other things. Some prostate glands normally produce more PSA than others. PSA levels tend to increase with age. And, PSA levels can vary with race: African Americans often have higher PSA levels; Asian men often have lower PSA levels. 38) Phlegmon is a. STD b. Types of cellulitis c. Types of osteomylitis d. Veneral disease ANS-(b) Type of Cellulitis (Shafers textbook of oral pathology 5th ed page no 697)

DENTAL QUEST Your Quest For The Success In Post Graduate Dental Entrance !

DENTAL

QUEST

AIPG -2011

15

CELLULITIS (PHLEGMON)

Diffuse inflammation of the soft tissues which is not circumscribed or confined to one area but which in contrast to the abscess, tends to spread through tissue spaces and along fascial spaces Spreading factors of Duran-Reynals o STREPTOKINASE, o HYALURONIDASE, o FIBRINOLYSINS.

Streptococci produce hyaluronidase and are a common causative organism than the staph. Clinical appearance o Orange peel.

A I P G 2 0 1 1

39) Fibrin degradation product help in detection of a. Hemophilia b. DIC c. Purpura d. Thrombasthenia ANS (b) DIC (Robbins & Cotran Pathologic Basis of Diseases 7th ed pg no 444-446) Fibrin degradation product help in detection of DIC 40) All of the following have defects in clotting mechanism except? a. Christmas disease b. Von willebrand disease c. Patients on brufen d. Thalassaemia Ans (d) Thalasemia (Ganongs Revive of Physiology 22nd ed page 545-545 and Katzung basic and clinical pharmoclogy 9th ed page no 1537) Christmas disease- absence of Factor IX Antihemophilic factor B or Christmas factor or PLASMA THROMBIPLASTIN COMPONENTPTC Von Willebrand disease- Compound defect involving platelet function and the coagulation pathway. Because Von Willebrand Factor stabilizes factor VIII by binding to it, deficiency of vWF gives rise to a secondary decrease in factor VIII levels. Ibuprofen is known to have an antiplatelet effect, though it is relatively mild and short-lived when compared with aspirin or other better-known antiplatelet drugs. (Katzung) Thalaseemia (Cooleys anemia, Mediterranean anaemia, erythroblastic anaemia)- Produces anemia and not defects in clotting mechanism. Thalassemic syndromes are genetically determined disorders of haemoglobin synthesis with decreased production of either alpha or beta polypeptide chains of haemoglobin molecules Surely, thalasemia does not affect clotting.

41)
a.

Myasthania gravis.which is not true Antibody formed to Ach receptors, b. Accumulation of Ach. at cleft c. Antibody bind to Ach. d. Anticholinesterases are used in the management Ans (b) accumulation of Ach at the cleft region (Shafers Textbook of oral Patholgy 5th ed page no 1183)

DENTAL QUEST Your Quest For The Success In Post Graduate Dental Entrance !

DENTAL

QUEST

AIPG -2011

16

Myasthenia Gravis is an autoimmune disease where the body develops antibodies to its own acetylcholine receptors.(so options (a) is right) The antibodies cross link the post synaptic nicotinic receptors at the neuromuscular junction. The antibodies either degrade this receptors or block them. Thus there is effective decrease in the number of receptors. ( so option c is right ) The acetylcholine release is normal at presynaptic membrane fold. The number of receptors available for effective depolarization is less. Muscular contraction is very slow and weak. Fatigue occurs quickly. In severe cases there is paralysis of muscles.

The acetyl choline at synaptic cleft however is degraded by cholinesterases accumulate at the synaptic cleft(so option B is wrong)

and wont

A I P G 2 0 1 1

The action of Ach can be potentiated by cholinesterase inhibitors ( Neostigmine and physostigmine) which prevent the degradation of acetyl choline (so answer d is right) 42) Leukemia is not common in which of the following syndromes a. Downs syndrome b. Blooms syndrome c. Turners syndrome d. Immune deficiency syndrome ANS (c) Turner syndrome (Harshmohans Textbook of General Pathology 3rd ed pg no 490) Except Turner syndrome other options shows increased frequency of leukemia 43) Heterozygous sickle cell anemia gives protection against a. G6PD b. Malaria c. Thalesemia d. Dengue fever ANS (b) Malaria (Robbins & Cotran Pathologic Basis of Diseases 7th ed page no 402,628) In a carrier, the presence of the malaria parasite causes the red blood cells with defective haemoglobin to rupture prematurely, making the plasmodium unable to reproduce. Further, the polymerization of Hb affects the ability of the parasite to digest Hb in the first place and hence Sickel cell anemia confers resistance against malaria in heterozygotes

MICROBIOLOGY AND IMMUNOLOGY


44) Bacteria causing Sub-acute bacterial endocarditis a. Strep. Viridians b. Staph aureus c. Lactobacillus d. Beta hemolytic streptococci Ans (a) Viridans streptococci (Kumar and Clarke textbook of Pathology 7th ed page no 829)

Viridans streptococci are normal oral flora. Streptococcus mutans comes under S. viridans (alpha-hemolytic) category. They produce dextran, a substance that allows them to adhere to many surfaces They are the major cause of subacute endocarditis (e.g., S.sanguis in those with abnormal heart valves. Streptococcus mutans causes dental caries.

DENTAL QUEST Your Quest For The Success In Post Graduate Dental Entrance !

DENTAL

QUEST

AIPG -2011

17

SABE Acute endocarditis Post operative endocarditis

Strep. sanguis (Strep, viridans group) Staph. aureus or pyogenes Staph. albus or epidermis

45) a.

Dengue haemorrhagic fever caused by Virus b. Mosquito Aedes aegypti c. Bacteria d. Ticks ANS (a) Mosquito Aedes aegypti (Textbook of Microbilogy by Anantanarayan 7th ed page no 532)

A I P G 2 0 1 1

Dengue haemorrhagic is caused by flavi virus of FLAVIVIRIDAE Aedes aegypti is a vector for Yellow fever, Dengue, Dengue hemorrhagic fever, Chikungunya fever, Rift valley fever. The Flaviviridae are a family of viruses that are primarily spread through arthropod vectors (mainly ticks and mosquitoes).The family gets its name from Yellow Fever virus, a type virus of Flaviviridae; flavus means yellow in Latin. They include the following genera: o Genus Flavivirus --Yellow fever virus, West Nile virus, Dengue Fever Japanese encephalitis, Tick-borne encephalitis o Genus Hepacivirus (type species Hepatitis C virus, the single member) o Genus Hepatitis G Virus (The GB agent and the hepatitis G virus (HGV)) o Genus Pestivirus (type species Bovine virus diarrhea, others include classical swine fever or hog cholera)contains viruses infecting non-human mammals 46) With which of following types of viral hepatitis infection in pregnancy ,the maternal mortality is the highest a. Hepatitis A b. Hepatitis B c. Hepatitis C d. Hepatitis E ANS-(d) Hepatitis E (Text Book of Microbiology by Ananthanarayan 7th ed page no 559) Maternal infection with hepatitis E often has an alarmingly high mortality, in the range of 2040%. 47) Donavanosis. True is, a. Caused by Calymatobacterium b. Painful ulcers c. It contains donavan bodies d. Spreads by blood transmission ANS (a) Calymatobacterium (Robbins and Cotran pathologic basis of diseas Volume 1,7th ed pg no 380) Granuloma inguinale or Donovanosis

Caused by Klebsiella granulomatis (formerly called Calymmatobacterium donovani). Organism is sexually transmitted Culture is difficult and diagnosis is mostly by microscopic examination

DENTAL QUEST Your Quest For The Success In Post Graduate Dental Entrance !

DENTAL

QUEST

AIPG -2011

18

Clinically extensive ulcerates which forms extensive scars Microscopic examination mimics pseudoepitheliomatous hyperplasia Donovan bodies-Encapsulated coccobacilli in macrophages. Silver stains like Warthin-Starry stain may be helpful Guys, looks like we have not found the right options. But hope these points are helpful. So mug this much and check in the exams.good luck. 48) All of the following are STDs except a. Herpes b. Scabies c. Candida d. Mycobacterium lepra Ans (d) Microbactrium Lepra (Sexually Transmitted Diseases by Sehgal1st ed page no 2-8)

A I P G 2 0 1 1

Leprosy is not a venereal (STD) disease. Leprosy or Hansen's disease (HD),is a chronic disease caused by the bacteria Mycobacterium leprae and Mycobacterium lepromatosis Usually spread from person to person in respiratory droplets. Leprosy is now known to be neither sexually transmitted nor highly infectious after treatment Sexually transmitted diseases (STDs) may spread through vaginal, anal and oral sex. Some infections, for example scabies, which are referred to as STDs because they are most commonly transmitted sexually,but which can also be passed on in other ways 49) Both CMI and HUMORAL IMMUNITY are best elicited by? a. Polysacchride Antigen b. Protein Antigen c. Lipid Antigen d. Adjuvant ANS-(d) Adjuvant (Textbook of Microbiology by Ananthnarayan 8th ed pg no 83,140)

The term adjuvant refers to any substance that enhances the immunogenicity of an antigen. Adjuvants may confer immunogenicity on non-antigenic substances, increase the concentration and persistence of the circulating antibody,induce or enhance the degree of cellular immunity. They also delay the release of antigen from the site of injection and prolong the antigenic stimulus. Viruses show both cell mediated and humoral response.Cell mediated immunity is common antibody in viral infections is IgM. Antigens when introduced into body induce antibody formation. However some antigens are involved in the sensitization of the Lymphocytes and production of cell mediated immunity. Antibody production is the property of B lymphocytes. For the full expression of this function, however, the cooperation of T lymphocytes is necessary. Depending on their ability to induce antibody formation, antigens are classified as T cell dependent (TD) and T cell independent (TI) antigens. Some antigens can directly stimulate antibody production by B cells, without the apparent participation of T cells, these are called as and T cell independent (TI) antigens. Other antigens require participation of the T-cells to elicit immune response. These are called as and T cell independent (TI) antigens Proteins are more antigenic than polysaccharides. Lipids and nucleic acids are less antigenic 50) Number of CD4 cells in a drop of blood in a normal person is

DENTAL QUEST Your Quest For The Success In Post Graduate Dental Entrance !

DENTAL a.

QUEST

AIPG -2011

19

500700 b. 400800 c. 7001000 d. 8001200 Ans (c) 700-1000 (Essentials of oral medicine by sol Silvermen 1st ed Page 132) Repeat MAHE-2009 In adults normal values for CD4 cell counts range from 800 to 1050 cells/cubic. mm

Normal People without HIV infection700 to 1000 CD4 HIV infected people are considered to have normal CD4 counts if the number is above 500 CD4 If the number of CD4 cells drops below 200 CD4 cells, it in AIDS.

A I P G 2 0 1 1

Normal CD4 percentages. These percentages refer to total lymphocytes. Normal range is between 20% and 40%. that means that 20-40% our lymphocytes were CD4 cells in normal healthy person. A CD4 percentage below 14% indicates serious immune damage. It is a sign of AIDS in people with HIV infection. A recent study showed that the CD4% is a predictor of HIV disease progression. Ratio of CD4 cells to CD8 The ratio of CD4 cells to CD8 cells is is calculated by dividing the CD4 value by the CD8 value. In healthy people, this ratio is between 0.9 and 1.9, meaning that there are about 1 to 2 CD4 cells for every CD8 cell. Opportunistic infections at the following CD4 levels

Less than 200: pneumocystis pneumonia (PCP) Less than 100: toxoplasmosis and cryptococcosis Less than 75: mycobacterium avium complex (MAC)

51) All of the following Statement about scrub typhus are true except a. Caused by O.Tstsugamushi b. Mites are act as reservoir c. Trasmitted when adult mites feed on hosts d. Tetracyclines is the drug of choice ANS (c) Transmitted when adult mites feed on host (PARK Textbook of Socai and Preventive Medicine 20th ed page no 262) Scrub typhus

Caused by Rickettsia tsutsugamushi (Orientatsu tsugamushi)- option a The true reservoir of infection is the trombiculid mite (Leptotrombidium delinese and L.akamushi). Chigger mites( larva) act as the primary reservoirs for O.tsutsugamushi- option b It is the larva (chigger) that feed on vertebrate hosts and picks up the rickettsiae. The larval stage serves both as a reservoir, through ovarian transmission, and as a vector for infecting humans and rodents. Transovarial transmission- They get infected abd they maintain the infection throughout their life stages and transform into adults, pass the infection on to their eggs in a process called.

DENTAL QUEST Your Quest For The Success In Post Graduate Dental Entrance !

DENTAL

QUEST

AIPG -2011

20

Transtadial transmission.- nfection passes from the egg to the larva or adult in a process The bite of the mite leaves a characteristic black eschar that is useful to the physicians for making the diagnosis. The larva (chigger) is the only stage that can transmit the disease to humans and other vertebrates, since the other life stages (nymph and adult) do not feed on vertebrate animals. (Hence Option C is wrong ) It has been shown that a single oral dose of chloramphenicol or tetracycline given every five days for a total of 35 days actually prevents against scrub typhus. This procedure is recommended under special circumstances in certain areas where the disease is endemic- Option d

A I P G 2 0 1 1

PHARMACOLOGY
52) The time taken for the drug to decrease by 50% of its concentration when the infusion is stopped is called as a. Context half life b. Elimination half life c. Elimination constant d. Resistribution half time Ans (a) Context half life (Clinical anesthesia by barsh-1st ed page no 156)

Context-sensitive half-life or context sensitive half-time is defined as the time taken for blood plasma concentration of a drug to decline by one half after an infusion designed to maintain a steady state (i.e. a constant plasma concentration) has been stopped. The "context" is the duration of infusion Remifentanil is relatively context insensitive whilst fentanyl and thiopentone are examples of drugs which have significant context-sensitive changes in their half-life. 53) Drug of choice in trigeminal neuralgiaa. Carbamazepine b. Phenytoin c. Veramamil d. Ethosuximide ANS-(a) Carbamazipine (Tripathis Essentials of Medical Pharmacology 6th ed page no 406 407) Repeat AIPG 2009

Carbamazipine is the first line of drug in Trigeminal neuralgia and often is used as confirmation for the diagnosis also Carbamazepine has become the drug of choice for trigeminal neuralgias. The most important action of carbamazepine is blockade of voltage-depending sodium ion channels. The most common adverse effects of carbamazepine are diplopia and ataxia. It may also cause aplastic anemia and agranulocytosis.

54) a. b. c. d.

Which of the following is true about Penicillin Acid resistant Excreted mainly through kidney Anaphylaxis never occurs more concentrated in prostatic secretions and intraocular fluid Ans (b) Excreted mainly through kidney

DENTAL QUEST Your Quest For The Success In Post Graduate Dental Entrance !

DENTAL

QUEST

AIPG -2011

21

( K D Tripathis Essentials Of Medical Pharmacology 5th ed page no 654)---REPEAT AIIMS NOV 09

A I P G 2 0 1 1

DENTAL QUEST Your Quest For The Success In Post Graduate Dental Entrance !

DENTAL

QUEST

AIPG -2011

22

55) Postural hypotension as first dose effect is seen in a. Prazosin b. Captopril c. Clonidine d. Nitroglycerine Ans-(a) Prazosin (Katzung pharmacology 9th ed page no 546)-- Repeat AIPG 2008

Postural Hypotension as first dose is seen in with prazosin which is a selective Alpha-1 receptor blockers but do not effect Beta -2 receptors. Alpha -1 receptor activation produces vasoconstriction, blockade of this by prazosin prevents vasoconstriction. However unopposed action of Beta -2 receptor activity leads to vasodilation > decrease in peripheral resistance> fall in B.P. Members of the prazosin family of alpha-1selective antagonists are efficacious drugs in the treatment of mild to moderate systemic hypertension. They are generally well tolerated by most patients. Their major adverse effect is postural hypotension, which may be severe after the first dose but is otherwise uncommon. Clonidine is a centrally acting alpla-2 agonist that causes more of neurogenic hypotension 56) Which of the following does not cause postural hypotension a. Prazosin b. Phentolamine c. Tamsulosin d. Ergotamine ANS-(c) Tamsulosin (K D Tripathis Essentials of Medical Pharmacology 6th ed page no 546)

A I P G 2 0 1 1

Tamsulosin possibly has less effect on blood pressure since it has some selectivity for alpha1A receptors. Tamsulosin is an alpha1 - adrenoceptor antagonist that was specially designed for the treatment of benign prostatic hyperplasia. Third- generation -adrenergic antagonists (eg, tamsulosin, silodosin) selectively block postsynaptic 1A-receptors, which concentrate in the prostate. 57) Which of the following is not an Alpha 2 blocker a. Yohimbine b. Phenoxybenzamine c. Phnotoalmine d. Prazosin Ans (d) Prazosin (K.D.Tripathi Essential of Medical Pharmocology 5th ed Page no 119 ) Classification of Alpha Adrenergic Blocking agents Nonequilibrium Type Beta-Haloalkylamines- Phenoxybenzamine

Equilibrium type (Competitive)


Nonselective

Ergot Alkaloids- Egotamine,Ergotoxine Hydrogenerated ergot alkaloid Dihydroergotamine, Dihydroergotoxine Imidazolines- Tolazoline,Phentolamine Miscellaneus-Cholropromazine,Ketanserin

DENTAL QUEST Your Quest For The Success In Post Graduate Dental Entrance !

DENTAL

QUEST

AIPG -2011

23

58) a. b. c. d.

Alpha-2 selective -Yohimbine Which of these drugs interacts with warfarin but doesnt increase INR OCP Metoclopramide Erythromycin Phneytoin

Alpha-1 Selective- Prazocin, Terazocin, Doxazocin

Ans (a) OCP (Textbook of General medicine by Clarke-5th-E-ed page no 567) OCP (oral contraceptive pills) decrease the efficiency of warfarin and decreases the INR Drugs /conditions that enhance potency and increased prothrombin time Disulfiram Phenytoin Metronidazole Eythomycin Trimethoprim Chloramphenicol sulfamethoxazole Broad spectrum Phenylbutazone antibiotics Aspirin Liver disease/. Thrombocytopenia/. Vitamin K Heparin deficiency/ Hypermetabolism Clofibrate Drugs that diminish potency and decreased prothrombin time Barbiturates Rifampin oral contraceptives ( remember RBC :- rifampicin, barbiturates, contraceptive) 59) True about ACE inhibitors a. Adverse effect is hypokalemia b. Secondary hyperaldosterism and K+loss due to diuretics c. Rebound hypertension on withdrawal d. Used safely in asthmatics ANS (d) Used safely in Asthematics (Pharmacology by Goodmann and Gillmann 11th ed page no 415)

A I P G 2 0 1 1

ACE inhibitors have no effect on heart rate or bronchioles, lipid levels or the action of insulin and therefore can be used safely in patients with heart failure, asthma, peripheral vascular diseases and diabetes

The attenuation of aldosterone production by ACE inhibitors (resulting hypoadosteronism) also influences K+ homeostasis. Substantial K+ retention can occur in patients with renal insufficiency. hyperkalemia may also develop when ACE inhibitors are used with other drugs that can cause K+ retention, including K+-sparing diuretics (amiloride, triamterene, and spironolactone), NSAIDs, K+ supplements Some patients with diabetic nephropathy may be at greater risk of hyperkalemia Rebound hypertension has not been reported with discontinuation of ACE inhibitors Which of the following is not a prodrug Quinapril Fosinopril Benzopril

60) a. b. c.

DENTAL QUEST Your Quest For The Success In Post Graduate Dental Entrance !

DENTAL

QUEST

AIPG -2011

24

d. Lisinopril ANS (D) Lisionopril (K D Tripathis Essentials of Medical Pharmacology 6th ed pg no 485) Most ACE inhibitors are prodrug which are converted by hepatic esterolysis to an active diacid metabolite Only captopril and lisinopril have sufficient oral bioavailability and are given as active drugs Propofol. All are true except

61)
a. b. c.

Has a rapid recovery rate Used for induction of anesthesia Causes vomiting after use d. Option not recalled(but surely not the ans) Ans (c) Causes vomiting after use (Katzung basic and clinical pharmacology 9th ed page no 599, 601 and Table 25-2 ) Repeate AIPG 2010

A I P G 2 0 1 1

Propofol is the most commonly used parenteral anesthetic. Propofol has rapid post-anesthetic recovery with complete clear headedness. The induction dose, onset, and duration of anesthesia are similar to thiopental. Because of its reasonably short elimination (t1/2) propofol often is used for maintenance of anesthesia as well as for induction. 62) Most important factor for calculating dose for NSAIDS in children? a. Age of child b. Body wt of child c. Flavour of salt d. Extent of pain ANS-(b) Body weight of Child (K D Tripathis Essentials of Medical Pharmacology 6th ed pg no 61)--Repeat AIPG -2009

Most drugs approved for use in children have recommended pediatric doses, generally stated as milligrams per kilogram or per pound. When pediatric doses are calculated, the pediatric dose should never exceed the adult dose Calculations of dosage based on age or weight are conservative and tend to underestimate the required dose. Doses based on surface area are also more likely to be adequate. 63) LA does not act in the presence of infection due to a. Increase in cationic form b. Increase in anionic form c. Presence of free base molecules d. High PH ANS (a) Increase in Cationic Form (MonhEims 7th ed page 129-131) Repeat AIPG 2010 LA are alkaloid bases that are combined with acids usually hydrochloric, to form water soluble salts. All anaesthetic salts are formed by a combination of a weak base & a strong acid. The salts are used because they are stable & water soluble; water solubility is necessary for their diffusion through interstitial fluids to the nerve fibres. In solution the salts of LAs exist in both free base form & as protonated charged molecules. The relative properties of each are decided by pKa of the salt. RN+H+ RNH+

DENTAL QUEST Your Quest For The Success In Post Graduate Dental Entrance !

DENTAL

QUEST

AIPG -2011

25

(free base) (protonated base) When injected, the relative proportions of each molecule species depends on Pka of salt and the tissue pH. When the tissue pH is low (as in inflammations, infections) the protonated cationic form increases while the free base form decreases. Since it is the free base form that penetrates across the nerve membrane, the increase in the cationic form limits the diffusion of LA across the nerve membrane lesser nerve blockade. This is the basis of LA being less effective in presence of pus/infection. 64) Less light headedness and early recovery and ambulation is due to a. Propofol b. Thiopentone c. N20 d. Halothene ANS (a) Propofol (Katzung page no 601 and 602)--Repeat Aipg-2009 Both Profpofol and Thiopentone have a very less induction time and rapid recovery time. But gyus in the textboo we also found that the Propofol is 10 times faster than thiopentals metablosim. Hence its better to go with option a 65) Intavenous Anesthetic agent with bronchodilator activity a. Propofol b. Halothene c. Ketamine d. Thiopentone Na ANS (c) Ketamine (Review of Clinical Anesthesia By Neil Roy Connelly -3rd/88 and KATZUNG Clinical Medicine 9th ed) Ketamine It has central sympathetic stimulatory effect hence bronchodilalation. Hence it is more useful for anesthetizing patients at risk for hypotension and bronchospasm and for certain pediatric procedures Ketamine has a bronchodilator action that may be mediated either via an increase in blood catecholamines or by its direct smooth muscle relaxant effect. In contrast to other parenteral anesthetics, ketamine increases cerebral blood flow and intracranial pressure (ICP) with minimal alteration of cerebral metabolism. Ketamine is relatively contraindicated for patients with increased intracranial pressure or those at risk for cerebral ischemia. Halothene is the preferred drug in asthamatics. It has also bronchodilating properties but is not an IV agent. 66) a. b. c. d. Which cephalosporin has anti pseudomonal action Ceftriaxone Cefaclor Cefotaxime Cefoperazone

A I P G 2 0 1 1

ANS-(d) Cefoperazone (K D Tripathis Essentials of Medical Pharmacology 6th ed pg no 796) Repeat AIIMS 2008 Ceftazidime,Cefopera zone and Cefpirome are active against Pseudomonas aeruginosa. 67) a. When patient has vancomycin resistant infection, tablet to be given is Ofloxacin

DENTAL QUEST Your Quest For The Success In Post Graduate Dental Entrance !

DENTAL

QUEST

AIPG -2011

26

b. Clarithromycin c. Cefatrime d. Linezolid ANS (d) Linezolid (K D Tripathis Essentials of Medical Pharmacology 5th ed pg no 691) Linezolid active against methicillin resistant and some vancomycin resistant Staph aureus (VSRA) Vancomycin is the drug of choice in MRSA and MRSE Newer drugs are developed to treat vancomycin resistant infections. Like inezolid (Zyrox),dolfropristin - quinupristin( synnercid), daptomycin( cubicin) 68) Which does not cause sedation a. Buspirone b. Diazepam c. Lorazepam d. Alprazolam ANS (a) Busprirone (K D Tripathis Essentials of Medical Pharmacology 5th ed pg no 401) Buspirone does not produce significant sedation or cognitive /fuctional impairment 69) Oral Contraceptive failure occurs in a patient on Rifampicin because of a. Rifampicin induce the metabolism of contraceptive b. Rifampicin simulates gonadotropin release from pituitary c. Rifampicin decrease the secretion of progestin d. Rifampicin antagonize the action of oral contraceptive ANS-(a) Rifampicin induces the metabolism of Contraceptives (K D Tripathis Essentials of Medical Pharmacology 5th ed pg no 700 Rifampicin induces the metabolism of Contraceptives by induction of liver enzymes hence there is a chance of Contraceptive failures It can be decreased by using an oral contraceptive containing higher dose of estrogen or by using alternative method of contraception. 70) Least efficient antiemetic drug a. Domperidone b. Cisapride c. Chlorpromazine d. Metaclopromide ANS (b) Cisapride (K D Tripathis Essentials of Medical Pharmacology 5th ed pg no 604 ) Cisapride is a prokinetic drug with little antiemetic property,because it lacks D2 receptor antagonism 71) a. b. c. d. All cause secondary polycythemia except High altitude Hemangioblastoma Myeloproliferative disorder Pheochromocytoma

A I P G 2 0 1 1

ANS-(c) myeloproliferative disorder (Pathophysiology of disease: an introduction to clinical medicine by Stephen 4th ed pg no 122)

Primary polycythemia Myeloproliferative disorder Secondary polycythemia High altitude

DENTAL QUEST Your Quest For The Success In Post Graduate Dental Entrance !

DENTAL

QUEST

AIPG -2011

27

o Hemangioblastoma o Pheochromocytoma 72) General anesthetic agent-- intrathecal application is contraindicated? a. Fentanyl b. Remifentanil c. Sufentanil d. Alfentanil Ans (b) Remifentanil The Essence of Analgesia and Analgesics (By Raymond S. Sinatra, Jonathan S. Jahr, J. Michael Watkins-Pitchford 1st ed page 149) Remifentanil

A I P G 2 0 1 1

Ultra short-acting synthetic opioid analgesic drug It is given to patients during surgery to relieve pain and as an adjunct to an anaesthetic.

Used for both sedation as well as combined with other medications for use in general anesthesia. Contraindications It conatins glycine which is present in the formulation. Glycine is an important inhibitory transmitter in the CNS In addition to opioid effects, it causes reversible motor weakness and hence remifentanil is not approved for epidural and intrathecal use. . -Fentanyl is a lipophilic opioid which has a fast onset of action when injected intrathecally -Intrathecal fentanyl, meperidine, and sufentanil alfentanil are given for labor analgesia.all this are synthetic opoid analgesics with Mu receptor activity.

ORAL ANATOMY AND HISTOLOGY 73)


Fibers attached to adjacent teeth a. Transeptal b. Apical c. Oblique d. Horizontal ANS-(a) Transseptal fibers (Carranzas Clinical Periodontology 10th ed pg no 70) Principle fibre of PDL are arranged in 6 groups Transseptal fibres :o Runs interproximally over alveolar bone to attached to cementum of adjacent teeth o Can again form after destruction of alveolar bone due to periodontal disease o Due to non osseous attachment they are also considered to be gingival origin

o o o

Alveolar crestal group :Runs obliquely from cementum beneath the junctional epithelium to the alveolar bone Prevent extrusion of teeth as well as resists lateral tooth movemnts Incision does not increase tooth mobility Horizontal group :Runs right angle to long axis of tooth from cementum to alveolar bone Oblique group :Largest group of fibres.

DENTAL QUEST Your Quest For The Success In Post Graduate Dental Entrance !

DENTAL
o o

QUEST

AIPG -2011

28

o o o o

Extend from cementum to bone in a coronal direction Faces the major part of vertical forces and convert them into tension on the alveolar bone Apical group :Extends in an irregular fashion in apical region of tooth socket Do not present in incompletely formed roots Inter radicular group :Present in furcation area Runs from cementum to tooth structure

A I P G 2 0 1 1

DENTAL MATERIALS
74) Back pressure porosity can be reduced by a. Vaccum investing b. Proper or increased burnout temperature c. Reduced firing temperature d. All ANS (b) Proper or increased burnout temperature (Craigs Restorative Materisls 11th ed page no 527) Repeat AIPG 2010 Back pressure porosity -Porosity produced in castings resulting from the inability of gases in the mold to escape through the investment. Methods to minimize the Back Pressure porosity Adequate mold and casting temperature. High casting pressure. Maintaining proper liquid: Power ratio during mixing of the investment. Practising a proper burnout technique during casting. The thickness of the investment between the tip of the pattern and end of the ring should not be greater than 6 mm in gypsum bonded investment and not more than 3.25 mm for a phosphate bonded investment. Using porous investment. Using burnout to eliminate wax which can block the pores. Using vents in a silica bonded investment.

ORAL PATHOLOGY
75) DNA repair defect associated with a. Xeroderma pigmentosa b. Icthyosis c. Mosaicism d. Turners syndrome ANS. (a) Xeroderma pigmentosa (Robbins & Cotran Pathologic Basis of Diseases 8th ed pg no 302, 103) Repeat AIPG 2010 Xeroderma pigmentosum Autosomal recessive or intermediate sex-linked inheritance. Genetic disorder of DNA repair in which the ability to repair is damaged by ultraviolet (uv) Especially prone to malignant melanoma. Three types of DNA repair defect contribute to different types of cancers o Mismatch repairs o Nucleotide excision repair

DENTAL QUEST Your Quest For The Success In Post Graduate Dental Entrance !

DENTAL
o

QUEST

AIPG -2011

29

Recombination repair

76)

Mosaic pattern of bone is seen in a. Fibrous dysplasia b. Osteitis Deformans c. Osteomylitis d. Ewings sarcoma ANS. (b) Osteitis deformans (Shafers Textbook Of Oral Pathology 5th ed page no 1005) Repeat AIPG 2010 One of most common features of Pagets disease or osteitis deformans is the formation of mosaic bone a descriptive term which indicates the appearance of bone which has been partially resorbed and then repaired leaving deeply staining hematoxyphilic reversal lines. 77) Root surface caries is caused by (repeated twice) a. Actinomyces viscosus b. Streptococcus viridians c. Streptococcus mutans d. Lactobacillus Ans (a) Actinomyces viscosus (Newburn earnst cariology 2nd page no 52 )

A I P G 2 0 1 1

For caries initiation (in both smooth surfaces and pit & fissures): Streptococcus mutans For caries progression (dentinal caries): Lactobacilli sp. For caries initiation on root surfaces: Actinomyces viscosus.

78) Which of the following is most likely fatal a. Osteosarcoma b. Gaint Cell tumor c. Paget disease d. Multiple myeloma ANS-(d) Multiple Myeloma (Shafers Textbook Of Oral Pathology 4th ed page no 181,191 & 5th ed pg no 276)

Multiple myeloma is a malignant neoplasm disorder. The prognosis is poor with the median survival time of 2-3 yrs Infection,anemia and kidney failure are common cause of death We found the word FATAL on 181 page when it is considered with other neoplasm

79) Herpangina is caused by a. Coxsackie virus b. Hepes zoster c. CMV d. HSV ANS-(a) Coxsackie virus (Shafers Textbook Of Oral Pathology 4th ed page no page no 375 ) Herpangina is caused by Coxsackie virus gr.A with types I through COX 6,8 10 16 and 22. 80) The deficiency of which of the following affects tooth development? a. Vit.A b. Vit.B c. Vit.E d. Carbohydrates ANS-(a) Vit A

DENTAL QUEST Your Quest For The Success In Post Graduate Dental Entrance !

DENTAL

QUEST

AIPG -2011

30

(Shafers Textbook Of Oral Pathology 5th ed page no 53) Vit A. effects growth of epithelial cells and hence tooth development Vit-D. is necessary for calcification Vit-C is essential for collagen formation.

81) Teeth that erupts within 30 days of birth are a. Neonatal b. Natal c. Primary d. Prenatal ANS.(a) Natal teeth (Mc DONALDS dentistry for child and adolescent 8th ed pg no 183) Neonatal teeth: those arising within the first 30 days of life are designated neonatal teeth. Clinical features: 85 %- mandibular incisors. 11 %- maxillary incisors. 4 % - posterior teeth Associated Syndromes Chondro ectodermal dysplasia Pachyonychia congenital Treatment : No hasty extraction since these might be deciduous teeth. If mobile = extract to prevent aspiration. Traumatic ulcerations of the adjacent soft tissue (Riga-Fede disease) may occur during breast-feeding but often can be resolved with appropriate measures. 82) a. b. c. d. Most common site for sialolith is Parotid duct Parotid gland Submandibular gland duct Sublingual

A I P G 2 0 1 1

Natal Teeth: teeth present in newborns have been called natal teeth.

ANS-(c) Submandibular gland duct (Shafers Textbook Of Oral Pathology 5th ed page no 754) Submandibular gland duct- Most common site for sialolith Reasn- Due to torturous wavy course of the duct,high mucin content and high viscosity of submandibular saliva, the stones are common in the gland

83) Which of following cause headache of vascular origin a. Giant cell arteritis b. Anaesthesia dolorosa c. Cluster Headache d. Hemicarnia ANS (a) Giant cell arteries (Burket's oral medicine 11t ed page no 294)

Temporal giant cell arteries or Giant cell arteritis is an inflammatory disorder of arteries that frequently involves extra cranial circulation Typical presenting symptoms include headache ,polymylagia rheumatic,jaw claudicating,fever and wt .loss HARRISONS

DENTAL QUEST Your Quest For The Success In Post Graduate Dental Entrance !

DENTAL

QUEST

AIPG -2011

31

84)

Most common malignancy seen in AIDS Kaposis Sarcoma b. Hodgkins tumor c. SCC d. Osteosarcoma ANS (a) Kaposi Sarcoma (Kumar and Clarks medicine 6th ed page no 142)

a.

The neoplastic diseases clearly seen with an increased frequency in patient with HIV infection are Kaposis Sarcoma and non Hodgkins LYMPHOMA 85) Mother donated one kidney to her daughter, what type of graft is it? a. Allograft b. Xenograft c. Cautograft d. Isograft ANS (a) Allograft Carranzas clinical periodontology 10th ed page no 978-80) Autograft- A transplanted organ or tissue from a genetically non-identical member of the same species. Isograft- A transplanted organ or tissue from a genetically identical donor, i.e. an identical twin, Allograft- A transplant from one person to another, but not an identical twin (Mother to daughter) Xenograft- a transplant from another species. Allografts and xenografts will be recognized by the recipient's immune system as foreign. 86) High viscosity of saliva is related to increased caries in children a. Really true b. Partially true c. Partially false d. Really false ANS-(a) Really True (Shafers Textbook of Oral Pathology 6th ed Page 427) The viscosity of saliva is largely due to its mucin content. Thick viscous saliva is more Cariogenic whereas copious thin Watery saliva has an excellentFlushing action and so less cariogenic 87) Degeneration of basement membrane and loss of rete pegs is seen in a. Lichen planus b. Desquamative gingivitis c. Erythema multiforme d. Leukoplakia ANS (a) lichen planus (Shafers Textbook of Oral Pathology 6th ed Page 802) Degeneration of the basal keratinocytes and disruption of the anchoring elements of the epithelial basement membrane and basal keratinocytes (eg, hemidesmosomes, filaments, fibrils) weakens the epithelial-connective tissue interface.This results in histologic clefts (ie, Max-Joseph spaces) and blisters on the oral mucosa (bullous lichen planus) may be seen at clinical examination. 88) a. b. c. What is seen in acute infection? Leucocytosis Leukopenia Lymphocytosis

A I P G 2 0 1 1

DENTAL QUEST Your Quest For The Success In Post Graduate Dental Entrance !

DENTAL

QUEST

AIPG -2011

32

d. Neutopenia ANS (a) leucocytosis (HARRISONS Internal medicine 16th ed page no 352) Repeat comed-2008 In most infections leucocytosis dominated by polymorhonuclear leucocytes (Neutrophils) are seen as they are the cells of first line defence. 89) Saw tooth rete pegs are seen a. A.Lichen planus b. B.Psoriasis c. C.Pemphigus d. D.Erythema multiforme ANS (a) Lichen Planus (Shafers Textbook of Oral Pathology 5th ed Page 1104) The microscopic appearance of lichen planus Hyperparakeratosis with thickening of the granular cell layer Development of a "saw-tooth" appearance of the rete pegs Degeneration of the basal cell layer with Civatte or colloid body formation. These result from degenerating epithelial cells. Infiltration of lymphocytic inflammatory cells into the subepithelial layer of connective tissue Wickham striae are whitish radiating lines visible in the papules of lichen planus and other dermatoses, typically the macroscopic appearance of the histologic phenomenon hypergranulosis, and named for Louis Frdric Wickham 90) A 6 yr old patient reports with greenish blue swelling distal to deciduous 2nd molar treatment would be a. No treatment ,only observation b. Surgical; excision of flap c. Bone graft d. Injecting proteolytic solution in area. ANS (a) No treatment,Only Observation (MC DONALD Dentistry for Child and Adolescent 8TH/ 182 183) Logical question.Clinical knowledge is a must for successfully attempting such type of questions. A 6 yr old patient reports with greenish blue swelling distal to deciduous 2 nd molar is eruption cyst which requires no treatment ,only observation 91) a. b. c. d. Which of the following seen with acromegaly Large tongue Micrognathia Crowding Hypogycemia

A I P G 2 0 1 1

Ans( a) Large Tongue (Shafers Textbook of Oral Pathology 6th ed Page 649) Soft tissue growth often produces uniform macroglossia in affected patients It causes Marcognathia , Spacing and Hypergylcemia

92)

A 6 yr old child presents with generalized ulceration of mouth ,malaise and fever. Treatment is a. Oral penicillin and peroxide mouth wash b. Prescribe broad spectrum antibiotics and mouthwash

DENTAL QUEST Your Quest For The Success In Post Graduate Dental Entrance !

DENTAL

QUEST

AIPG -2011

33

c. Gently debride the ulcers and symptomatic treatment only d. Systemic antibiotics only ANS ( c) Gently debride the ulcers and symptomatic treatment only (BURKETS ORAL MEDICINE 11th ed page no 46)

In above case the the symptoms relate to the Herpetic gingivo stomatitis.

Acute herpetic gingivostomatitis patients who have generalized ulceration of mouth ,malaise and fever- Gently describe the ulcers and symptomatic treatment should be given Oral pencilllin and hydrogen peroxide mouthwash is recommended in ANUG Radiographically the level of alveolar crest corresponds to? CEJ of adjacent teeth Dentoenamal junction Trabeculae pattern Gingival margin

A I P G 2 0 1 1

93)

a.
b. c.

d.

Ans (a) CEJ of adjacent teeth (CARANZAs Clinical Periodontology 10th ed page no 561 & White and Pharoah Oral Radilogy 5th ed pg no 317) Under ideal conditions, the crest of the interdental bone should be located not more than 1-2 mm apical to the cementoenamel junction (CEJ) 94) Most common tumor of parotid gland a. Pleomorphic adenoma b. Adenoid cystic carcinoma c. Warthins tumor d. Mucoepidermoid carcinoma Ans(a) Pleomorphic Adenoma (NEVILLE Oral Pathology 2nd ED page no 410-413) Pleomorphic adenoma OR BENIGN MIXED TUMOR IS MOST COMMON salivary tumor Most common maliganant tumor of salivary glands is is mucoepidermoid carcinoma

95)

Turner hypoplasia is caused by a. Hypoplasia due to local trauma b. Hypoplasia due to systemic infection c. Hypoplasia due to congenital syphilis d. Hypoplasia due to fluoride ingestion Ans (a) Hypoplasia due to local trauma (Shafers Textbook of Oral Pathology 5th ed page no 72-73) Turner hypoplasia is caused by Hypoplasia due to local trauma or infection Turners tooth of central incisor in seen in congenital syphilis

ORAL SURGERY
96) Most common cause of TMJ ankylosis is: a. Trauma b. Developmental disturbances c. Infection d. Atrophy ANS-(a) Truama (Textboof of Oral Surgery by N.Malik -207 and V.Kapoor-206) Guys this is a frequently repeated question

DENTAL QUEST Your Quest For The Success In Post Graduate Dental Entrance !

DENTAL 97)

QUEST

AIPG -2011

34

Truama results in extravasation of blood into joint space. This predisposes to calcification and finally results in ankylosis

After obtaining an incisional biopsy,a negative report in the face of a clinically suspicious lesion means a. Second biopsy should be taken b. The patient should be observed for one month c. The patient can be assured of no malignancy at this time d. The patient should undergo yearly observation of the area ANS-(a) Second biopsy should be taken (NBDE PART-II oral surgery and pain control march 1983 /Q no.26)

A I P G 2 0 1 1

A Second biopsy should be taken in suspected cases Another biopsy is necessary in view of the clinical impression One should give priority to the clinical impression. Investigations usually assist the physician to come to the definite diagnosis

98) Which is not injured in lower third molar extraction? a. Inferior alveolar nerve b. Inferior alveolar artery c. Lingual nerve d. Lingual artery Ans (d) Lingual Artery (Peterson's principles of oral and maxillofacial surgery,2nd ed volume 2 page no 820) Again a clinically oriented question nyways let us explain this too.

Lingual artery by elimination is the correct answer. Usually the Lingual artery is not in close proximation to the third molar area compared to the other three. It runs deep to to hyoglossus muscle in the floor of the mouth. It is injured in cases that cause laceration of the soft tissue of the floor of the mouth deep at the root of the tongue. The overall risk of inferior alveolar nerve injury with third molar removal ranges from 0.5% to 5%. The incidence of lingual nerve injury is considerably lower than for inferior alveolar nerve injury and ranges from 0.02% to 0.06%.

99)

X ray for third molar reveals that inferior nerve canal becomes thinner along the roots of third molar.The most possible cause is? a. Root perforated by nerve, b. Nerve goes between root of teeth c. When canal crosses the root apex d. Nerve encircles the root Ans- (c) When canal crosses the Root Apex Textbook of Oral and maxillofacial surgery by N Mallik - 2nd ed page no 129.) Question are being frequently repeated on this topic.

Converging or narrowing of canal occurs When canal crosses and superimposes the root apex there is reduction in its diameter. Pre operative assessment was carried out radiologically in an attempt to identify the proximity of the impacted tooth to the inferior alveolar canal. The important radiological signs, four of which are related to the root of tooth and three signs were related to the appearance of inferior alveolar canal. o Related to tooth

DENTAL QUEST Your Quest For The Success In Post Graduate Dental Entrance !

DENTAL

QUEST

AIPG -2011

35

Darkening of root was due to decreased amount of both tooth substance or loss of cortical lining of the canal between source of x-rays and film. Deflected roots were due to deviation of the root to the buccal or lingual sides or both, and even to the mesial or distal aspect. Narrowing of root this means that the greatest diameter of root has been involved by the canal or there is grooving or perforation of the root Dark and bifid root appeared when inferior alveolar canal crossed the apex of the root, this was identified by double periodontal membrane shadow of the bifid apex. Related to canal Interruption of the white line this appeared due to dense structure of canal walls Diversion of inferior alveolar canal- happened when it crossed the mandibular third molar. It changed its direction Narrowing of inferior alveolar canal this happened when it crossed the root of mandibular third molar. Hour Glass appearance indicates partial or total encirclement of the canal.

A I P G 2 0 1 1

100) Eutectic mixture of 2.5 % Lidocaine and 2.5% Prilocaine primarily used to a. Gasserion ganglion block b. Gow gates inferior aloveolar nerve block c. Topical skin application d. Spinal anesthesia Ans (c) Topical skin application (Local Anesthesia by Malamed 4th ed page no 288) EMLA (eutectic mixture of local anesthesia) consists of a 5% cream containing 25mg/g of lidocaine and 25 mg/g of prilocaine.

It has been shown to provide anesthesia of intact skin profound enough to permit venipuncture to be performed painlessly. It is applied to the skin for at least 1 hour before the anticipated surgery. Requirement of EMLA in : Venipuncture, Pediatric dentistry, Vaccination, Suture removal, Lumber puncture, Minor otological surgery, Minor gynecological and urological Procedure,dermatological surgery and debridement of infected ulcers, etc.

101) Excisional biopsy is done by a. Removal of entire lesion only b. Removal of entire lesions with a margin of uninvolved normal border of tissue c. Removal of only a part of lesion d. Partial removal of lesion with part of normal tissue Ans (b) Removal of entire Lesion with a margin of uninvolved of normal tissue. (Petersons 3rd ed page no 521) Repeate AIPG 2009 Excisional biopsy is done by complete excision of the tumor along with some part of normal tissue Excisional biopsy. Complete excision of the tumor along with some part of normal tissue A special type of excisional biopsy of the breast is the needle localization biopsy, also called the "wire-guided biopsy." Incisional biopsy Only a portion of the lump is removed surgically.

DENTAL QUEST Your Quest For The Success In Post Graduate Dental Entrance !

DENTAL

QUEST

AIPG -2011

36

Most commonly used for tumors of the soft tissues Endoscopic biopsy It is done through a fiberoptic endoscope Colposcopic biopsy The colposcope is actually a close- focusing telescope Fine needle aspiration (FNA) biopsy. Very simple Needle of (22 to 25 gauge) is inserted into (tumor), few cells are aspirated into a syringe. no scar, no anesthesia, inexpensive Stereotactic needle biopsy Relatively new technique mainly for breast lesions Combines the advantage of advantages of FNA and excisional biopsy (collection of solid pieces of tissue rather than smears) and needle localization (precise guidance by x-ray or ultrasound imaging). Punch Biopsy Mainly for skin lesions Bone marrow biopsy Used in cases of abnormal blood counts 102) The biomechanics of alveolar process during fractures at the angle of mandible is a. Compressive force b. Tension force c. Torsional force d. Twisting force Ans (b) Tension Force (Rowe and Williams maxillofacial injuries Vol 1 pg no 6) Repeat AIIMS nov 2008

A I P G 2 0 1 1

Positive bending moments are those moments which causes compression at the lower border and tension at the alveolar side of the mandible Negative bending moments are those moments that cause compression at the alveolar side and tension at the lower border of the mandible. Angle, body and symphyseal fractures have a characteristic load pattern. These load patterns play a decisive role in the treatment of mandibular fractures with regard to number and positioning of plates. Angle fractures had relatively high positive bending moments. Compression at the lower border and tension at the alveolar side of the mandible. Body fractures had positive as well as negative bending moments and the highest torsion moments. Symphyseal fractures had negative bending moments only and relatively high torsion moments

103)

Patient complains of pain and foul smell on third day after extraction. Treatment is a. Irrigate the socket and place sedative dressing b. Sedative dressing c. Search for the root pieces d. All of the above Ans (a) Irrigate the Socket and Place Sedative Dressing (Peterson oral surgery 2nd ed page no 234 and direct pick from NBDE part-I) The symptoms may be dur to dry socket Dry Socket also known as o Alveolitis sicca dolorosa o Alveolalgia o Localized acute alveolar osteomyelitis

DENTAL QUEST Your Quest For The Success In Post Graduate Dental Entrance !

DENTAL
o o

QUEST

AIPG -2011

37

Alveolar osteitis Postoperative osteitis

Clinical features Most common and Throbbing severe painful complication after pain extraction No swelling Foul odor No suppuration Focal osteomyelitis present More common in o females, o Tobacco users o After traumatic extraction o In Pagets diseases o Patient under radiotherapy o Patients taking oral contraceptives- estrogen increases fibrinolytic activity

A I P G 2 0 1 1

Frequency 1-3 % but 20% at third molar extraction cases Starts on 2nd or 3rd day after extraction

According to Nitzin's theory, Treponema denticolum is responsible for dry socket Management- irrigation of socket with warm saline followed by obtundant dressing.

RADIOLOGY
104) Best radiographic film to detect incipient caries is a. Speed E film b. Speed D film c. Speed F film d. Speed C film Ans is (b) D film (Sturdevant Art and Science of Operative Dentisry 4th ed page no 105) Repeat: Kar 2008, AIIMS may 2009

Guys apart from logical reasoning of contrast and detection of incipient caries wrt film D and E, we also found this line in Sturdevants There is one study which reported that the newer film have less contrast and have less ability to detect caries than the newer films specilly in the proximal lesion

But the question might be different... so we would like to tell you another thing. Both the film has equal efficiency as far as other books are concerned. Use speed E and speed F film for incipient caries when it comes to best for patient aspect

But when it comes to best for only for the detection of incipient caries then the answer will be D film we guess its better to go with the D in the above question.

DENTAL QUEST Your Quest For The Success In Post Graduate Dental Entrance !

DENTAL

QUEST

AIPG -2011

38

105) Radiographic features of sinusitis includes a. Fluid level b. Erosion of bone c. Clouding of antrum d. Fluid level and clouding Ans is (d) Clouding and Fluid Level (Whites and Cawson Dental radiology 3rd ed page no 339)- Repeat AIPG 2009 and Also in national boards Thickening of sinus mucosa and accumulation of secretions in sinusitis reduce the air content of the sinus and cause it to become increasingly radiopaque. Irregular increase in opacity on the inner aspect of the walls owing to the mucosal thickening. This basically explains clouding Fluid levels in the antrum in the shape of the meniscus also found. This is confirmed by, a second occipitomental view which should be taken with the patients head tilted to one side. The fluid level in the antrum should then change correspondingly. The radiographic appearance of a fluid level is so characteristic that this is seldom necessary. Radiolucent air shadow evident in the central part of the antral cavity can also be found 106) Digital fibre optic transillumination(DFOTI) is mainly used to: a. Determine the bone loss b. Determine the proximal caries c. Determine the calcium concentration d. Determine the occlusal caries of enamel Ans (b) Determination of Proximal Caries ( Mc Donalds Dentistry for the Child and Adoloscence 8th ed page no 216) 2008 (One of the most repeated questions in recent entrances)

A I P G 2 0 1 1

Repeat AIIMS nov

DIGITAL IMAGING FIBER-OPTIC TRANS-ILLUMINATION (DIFOTI) intense light beam that is transmitted through a fiberoptic cable to a specially designed probe to permit The use of transillumination on the proximal surfaces of posterior teeth. DIFOTI system, unlike radiography, was not able to determine the depth of lesions. Also remember diagnodont is mainly used for occlusal caries where as DIFOTI can be used to detect both proximal (mainly) and occlusal caries.

PERIODONTICS
107) 'Indifferent' fibers are a. Elastic fibers b. Oxytalan fibers c. Collagen fibers d. Reticulin fibres Ans (c) Collagen Fiber (Carranzas Clinical Periodontolgy 9th ed page no 38 ) PDL from principal fibres containing small collagen fibers associated with the large principal collagen fibers have been detected .These fibers run in all directions from a plexus called the '' indifferent fiberplexus''. Scanning electron micrographs demonstrate the presence of an indifferent fiber plexus as an integral part of periodontia in the rat, dog, armadillo and man. Individual fibers measure 1400 in diameter in scanning electron micrographs. The indifferent fibers course in every direction throughout the area between bone and cementum without particular orientation. They anastomose extensively with the principal fibers to form a continuous fibrous matrix. Other points

DENTAL QUEST Your Quest For The Success In Post Graduate Dental Entrance !

DENTAL

QUEST

AIPG -2011

39

Collagen fibres in PD ligament are predominantly type I.


Oxytalan fibres and Eluanin are immature elastic fibres which extend from cementum to blood

vessels and supports blood vessels. Reticulin fibres are class III collagen fibresimmamture collagen fibres Mature elastic fibres are usually not found in the PDL ligament. 108) Which organism is not seen in periodontosis? a. Mycoplasma b. Actinomyces c. Actinomycetamcomitans d. P.Intermedia ANS (a) Actinomyces (CARRANZAS CLINICAL PERIODONTOLOGY 10th ed page no 509-510) Rest all organism are found in periodontosis or Aggressive Periodontitis or Juvenile Periodontitis.

A I P G 2 0 1 1

110) In the soft tissue wall of the periodontal pocket,what type of changes are seen

a. b. c.

Proliferative changes are seen. Degenerative changes are seen. Both proliferative and degenerative changes are seen. d. No changes are seen. ANS (c) Both proliferative and degenerative changes are seen. (Caranzas clinical periodontology 10th Ed Pg no 438) Repeat of AIPG 2010 Soft Tissue Wall Changes in Periontal Pocket Infiltrated with around 80% plasma cells Shoorter junctional epithelium reduced to 50 to 100 m Severe changes in lateral walls Has both Proliferative and degenerative changes In aggressive and adult periodontitis more pronounced degenerative changes with more open intercellular spaces and microcleft and necrotic area.

111)

The severe alveolar bone loss in patients with juvenile periodontitis is associated with a. impaired osteoblastic activity. b. Increased phagocytosis. c. Impaired neutrophil chemotaxis. d. Increased macrophage migration. ANS (c) Impaired neutrophil chemotaxis (CARRANZAS CLINICAL PERIODONTOLOGY 10TH ed pg NO 244) There is phagocyte abnormality Dysfunction of neutrophils The neutrophil defect is called as Global Membrane defect

112) A young child of 7 years age is seen with indurated ulcers, lymphadenopathy and fever,
the likely (diagnosis and) treatment will be a. Symptomatic treatment b. Systemic antibiotics c. Excision of the lesion d. I.V. fluids Ans (a) Symptomatic Treatment (CARRANZAS CLINICAL PERIODONTOLOGY 10TH ed page no 771) Repeat AIPG

DENTAL QUEST Your Quest For The Success In Post Graduate Dental Entrance !

DENTAL

QUEST

AIPG -2011

40

Guys... Note the point 7 years, Fever,and Ulcers.These all indicate that this is a case of herpetic Gingivostomatitis as herpetic lesion is more common in infants and children of more than 6 years.

Now for this question its better to go with old edition of Caranza. But you will surely find a line in 10th ed which says that the earlier protocol was only palliatve treatment. But what is important to mention here is, in new edition the author clearly says that if the herpetic gingivitis in diagnosed within 3 days of onset then Acyclovir should be given 15mg/kg five times daily for 7 days in a suspension form. So remember this point

A I P G 2 0 1 1


113)

For those who present after 3 days of onset then palliative treatment should also be given

Also remember that local and systemic antibiotic are given just prevent secondary infection. So option B not preferred for sure Anyway there is no option of antiviral drugs in above question.So option a is best to choose.

Periodontometer is used to detect a. Tooth mobility b. Periodontal pocket c. Dental caries d. Truma from Occlusion Ans (a) Tooth Mobility (Glickmans Clinical Periodontology 7th ed page no 483) Repeat AIPG 2010 These are mechanical or electrical devises to accurately measure tooth mobility. 114) Apical migration of epithelial attachment with corresponding recession of marginal gingiva results in a. Shallow sulcus b. Gingival pocket formation c. Infrabony pocket formation d. Periodontal pocket formation ANS-(a) shallow sulcus or (d) Periodontal pocket formation (Carranzas Clinical Periodontology 9th ed page 336) Actually both A' and 'D' appear to be correct.

But guys Answer D is appropriate, though its being lifted from a book which seems to be a question bank of aipg examiners where answer is given as SHALLOW SULCUS and we suggest you to choose options carefully. Pocket depth is distance between gingival margin to the base of the pocket[or coronal end of junctional epithelium]

Periodontal or True Pocket can be defined as deepening of the gingival sulcus with loss of attachment.( So option D is appropriate) If there is loss of attachment of more than 3mm, a sulcus is technically called as pocket Depending upon the relation of the base of the pocket to the crest of the alveolar bone , it may be supraalveloar or infra alveolar. In supraalveolar bone loss is Horizontal where

DENTAL QUEST Your Quest For The Success In Post Graduate Dental Entrance !

DENTAL

QUEST

AIPG -2011

41

as in infraalveolar it is vertical which can be diagnosed on radiographs. In this question this doesnot arise ( option C may not be considered)

Coronal migration of the marginal gingiva gives rise to gingival pocket formation( so option B: can be eliminated) But if gingival recession is there, it may give rise to decrease in the depth of the pocket formed, so appropriate answer would have been periodontal pocket with less depth. After attachment loss a sulcus is converted into a pocket. Pocket depth is less important than loss of level of attachment because it is not necessarily related to bone loss. A tooth with deep pockets may have little bone loss while a tooth with shallow pockets may have severe bone loss. Prognosis is considered as poor if the base of the pocket [level of attachment] is close to root apex. 115) a. b. c. d. Gingival clefts may be formed by Occlusal disharmonies Faulty tooth brushing Normal frenum attachment Use of dental floss

A I P G 2 0 1 1

Ans (b) Faulty tooth brushing (Glickman textbook of Periodontics -8th page no 124 and directly taken from national Boards) Repeat AIPG 2009 Gingival Cleft A narrow type of gingival deformity encountered on the marginal aspect of the gingiva and extending regularly or irregularly from the gingival margin in an apical or lateral direction, with margins that are usually thick and margins which are rolled inside

A cleft of the marginal gingiva; may be caused by incorrect toothbrushing, Pocket formation, or faulty tooth positions, and may resemble a V-shaped notch.

The clefts are prominent on the tooth with thin labial cortex such as mandibular canines and premolars. They are also seen in malpositioned tooth. They often are caused due to faulty tooth brushing (particularly FONEs technique). They are also seen in with hard brush or a change in new brush. They disappear once the brushing technique is corrected. Stillman clefts are apostrophe shaped indentations extending to marginal gingiva. They are usually present on facial surface. Mccalls festoons are Life saver shaped enlargement of marginal gingiva. Both Stillmans clefts and Mccalls festoons are considered to be the result of trauma from occlusion. (Remember this) The interdental clefts are normal anatomic features found in the interradicular zones underlying the saddle areas.They were attributed to trauma from occlusion, but this association was never proven (Remember this also );they represent peculiar inflammatory changes of the marginal gingiva. 116) Tobacco chewing is thought to be associated with which of the following conditions ? a. Desquamative gingivitis b. ANUG c. Juvenile periodontitis d. Erythema multiforme ANS (b) ANUG (Carranzas Clinical Periodontology 10th ed page no 391)

DENTAL QUEST Your Quest For The Success In Post Graduate Dental Entrance !

DENTAL

QUEST

AIPG -2011

42

ANUG is characterized by sudden onset of symptoms,sometimes following an episodes of debilitating diseases or acute respiratory tract infections. A change in living habits,protracted work without inadequate rest ,poor nutrition ,tobacco use and psychological stress are frequent features of patient history The correlation between tobacco smoking and acute necrotizing ulcerative gingivitis (ANUG) has been clearly shown. The cause and effect relation is not directly established. Both smoking and ANUG are the result of anxiety and tension. 117) Attached gingiva is a. Always stippled b. Non keratinized c. Resistant to masticatory stress and forces d. Resistant to inflammatory changes ANS-( c) Resistant to Mastricatory stress and forces (Carranzas clinical periodontlogy10th ed pg 47, 62 ,504)

A I P G 2 0 1 1

Attached gingiva extends from free gingival groove to Mucogingival line. It is always normally stippled. Stippling may be absent in infants and old age. It is always keratinised.

118) Type of bone present in inter-radicular area is predominantly a. Cortical b. Cancellous c. Osteophytic d. Exophyri ANS (b) Cancellous bone (Carranzas Clinical periodontology 9th ed pg 48) Repeat AIIMS nov 2006 Cancellous bone found predominantly in the inter-radicular and interdental spaces 119) Most common cause of dry mouth in adults a. Tarnquilizers b. Anti-histaminics c. Insulin d. Birth control pills Ans (b) Antihistamines ( Reff Burkitt s oral medicine 10th ed page no 210 and Shillingburgh 3rd ed pg no 4) Repeat AIPG 2009 The order of group of drugs given in the table are Anticholinergics, antihistaminics, antihypertensives, Antiparkinsonism, Anti seizure Sedatives and Tranquillizers Skeletal muscle relaxants Ticyclic anti depressents Anti histaminics comprises the largest group of such drugs and chronic allergy sufferers who use them over a prolonged period of time may suffer from a dry mouth (Shillingburgh) 120) Common clinical sign of occlusal trauma a. Tooth mobility b. Fractures of cusps(wear facet) c. Resorption of alveolar ridge

DENTAL QUEST Your Quest For The Success In Post Graduate Dental Entrance !

DENTAL

QUEST

AIPG -2011

43

d. Widening of PD ligament Ans (a) Tooth Mobility (Carranzas Clinical Periodontology 9th ed pg 377 and 10th ed 474) Repeat -AIPG 97, 89 , MAHE-98, AIIMS -92, comed-2007 Most common clinical sign of trauma from occlusion is tooth mobility. Resorption of bone and widening of PD ligament are radiological features. Fremitus test is done to test the mobility of teeth Increased tooth mobility is the most common clinical sign of truma from occlusion Probable etiology of gingivosis is a. High progesterone levels b. Deficiency of estrogen and testosterone c. Pregnancy d. Aldosterone deficiency ANS-(b) Deficiency of estrogen and Testosterone (Glickman textbook of Periodontology 7th ed page no 174) Repeat AIPG 2009 Chronic desquamative gingivitis also called gingivosis. Hormonal imbalance, deficiency of estrogen in the female or testosterone in the male and nutritional deficiency is suggested etiologic factors. 122) Supragingival plaque undergoes which of the following changes with time. a. Plaque mass decreases b. Plaque microflora becomes more gram positive c. Plaque microflora becomes more gram negative d. Plaque microflora becomes predominantly spirochetal ANS-(c) Plaque Microflora becomes more gram negative (Carranza 8th edn pg no 89 and 9th ed pg no 101) Early supragingival plaque contains gram +ve cocci and rods while mature supragingival plaque contains gram ve rods and filaments 123) Difference in the colour between sub and supragingival calculus is related to a. Ph of saliva b. Death of leukocytes c. Hemolysis of erythrocytes d. All of the above ANS (c) Hemolysis of Erythrocyte (Carranzas Clinical Peridontology 9th ed page no 98) Repeat AIPG 2009 Subgingival calculus is typically dark green or dark brown, probably reflecting the presence of subgingival matrix components distinct from those of Supragingival calculus (eg. Blood products associated with subgingival Heamorrhage ) 121)

A I P G 2 0 1 1

124) Which of the following is a characteristic of supragingival plaque and not of


subgingival plaque in humans a. Motile bacteria are predominant b. Spirochetes are evident microscopically c. Gram negative bacteria are predominant d. Bacterial composition is altered by dietary sugar consumption ANS (d) Bacterial composition is altered by dietary sugar consumption) (Carranza Clinical Peridontology 8th ed pg no 85 & 9th ed pg no 98) Repeat AIPG 2009 Plaque contains about 5% of inorganic components.

DENTAL QUEST Your Quest For The Success In Post Graduate Dental Entrance !

DENTAL

QUEST

AIPG -2011

44

The principal inorganic content of plaque matrix are calcium and phosphorus .Other elements are magnesium, potassium,sodium and fluoride. flouride deters the metabolism of plque bacteria and aids in remineralisation of tooth surface The source of inorganic content of supragingival plaque is saliva and for Subgingival plaque is GCF 125) Delayed hypersensitivity or cell mediated immune reactions occur in patients with periodontal diseases because they often have a. Igg antibody reactine with plaque bacterial antigens b. T Lymphocytes sensitized to plaque bacterial antigens c. Soluble immune complexes within involved gingival tissues d. All of the above ANS.-(b) T-Lymphocytes Sensitised to Plaque Bacterial antigens (Carranzas Clinical Periodontology 8th Ed Pg 118 & 10th Ed Pg 222) Repeat AIPG 2009 and National Boards Cell mediated immune reactions or delayed hypersensitivity dose not involve antibodies but is based on the interaction of antigens with surface of T-Lymphocytes 126) A radiograph of the mesial aspect of the mandibular right first molar shows 23 mm of bone loss, yet clinical probing indicates a pocket depth of 68 mm.This discrepancy is probably due to the? a. Poor angulation used in taking the radiograph b. Presence of mandibular tori in the area c. Presence of mesial bone masking the destruction d. Presence of facial or lingual bone masking the destruction Ans is (c) Presence of Facial and Lingual Bone Masking the destruction (Carranzas Clinical Periodontology 9th ed page no 456)

A I P G 2 0 1 1

Radiographs do not reveal the extent of involvement of bone loss on the facial and lingual surfaces. Facial and lingual surface bone destruction is obscured by the dense root structure, and bone destruction on mesial and distal root surface may be partially hidden by a dense mylohyoid ridge. The true lesion can only be detected by clinical probing. For destruction of the interproximal cancellous bone to be recorded radiographically,the cortical bone must be involved. A reduction of only 0.5 to 1.0 mm in the thickness of cortical plate is sufficient to permit visualization of destruction of the inner cancellous trabeculae. 127) Dental Floss should ideally be used in Type I embrassures Type II embrassures Type III embrassures d. All of the above ANS-(a) Type I embrassures (Carranzas Clinical Periodontology 10th ed page no 735-737)

a. b. c.

Dental floss ---Type I embrassures Unitufted brushType III embrassures

128) Which of the following is most useful in differentiating an acute periodontal abscess from periapical abscess a. Type of exudate b. Nature of swelling c. Intensity of pain d. Result of periodontal probing

DENTAL QUEST Your Quest For The Success In Post Graduate Dental Entrance !

DENTAL

QUEST

AIPG -2011

45

ANS-(d) Result of Periodontal Probing (Carranzas Clinical Periodontology 8th Ed Pg no 630 & 10th Ed Pg no 557) Repeat AIPG 2009 Periodontal abscess usually occurs in a pre existing pocket Probing will often identify a communication between the gingival margin and the abscess area 129) Which of the following indicies is used to assess the severity of periodontitis in epidermiological studies of a large population? a. PMA index b. Gingival index c. Periodontal index d. Sulcus bleeding index Ans is (c) Periodontal Index (Textbook of Community and Preventive dentistry by Soben peter 2nd pg no 159) The periodontal index is reported to be useful among large populations, but it is of limited use for individuals or small groups.The periodontal index is probably the most widely used periodontal index in epidemiological surveys around the world .It was developed by RUSSEL in 1956. 130) Which of the following plaque indices does not use disclosing agent a. Plaque component of periodontal disease index b. Debris component of Simplified Oral Hygiene Index c. Modified Quingley Hein plaque index d. All of the above Ans(b) Debris Component of Simplified oral Hygine index (Textbook of Community and Preventive dentistry by Soben peter 2nd ed page no 136,164) Repeat MAHE -2008

A I P G 2 0 1 1

The Oral Hygiene Index (OHI) was developed in 1960 by John C. Greene and Jack R. Vermillion to classify and assess oral hygiene status of population. Only a mouth mirror and a shepherd's crook or sickle type dental explorer, and no disclosing agent, are used. This can be modified for personal use by using disclosing agents. The disclosing agent used in modified quigley hein plaque index is Basic fuchsin. PDI index by Ramjford utilizes Bismarck Brown. 131) Following plaque index divides each tooth surface into 9 areas a. PHP index(Patient hygiene performance index) b. Plaque index by silness and loe c. Modified Navy plaque index d. Glass criteria for scoring debris Ans (c) Modified Navy Plaque Index (Carranzs Clinical Periodontology 8th ed page no 69) Repeat AIPG 2009 The modified Navy plaque index records the presence or absence of plaque with a score of 1 or 0, respectively, on nine areas of each tooth surface of the six index teeth use by Ramjford. 132) In gingivitis, the role of immunoglobulins is consistent with increased number of a. Fibroblasts b. Neutrophils c. Lymphocytes d. Plasma cells Ans (d) Plasma cells (Caranzas Clinical Periodontology 10th ed page no 241)

DENTAL QUEST Your Quest For The Success In Post Graduate Dental Entrance !

DENTAL

QUEST

AIPG -2011

46

The proportion of lymphocytes to plasma cells varies with the level of inflammation present in the tissues. In early stages T-lymphocytes dominate where as in established cases of gingivitis, plasma cells/B-cells dominate. The immunoglubulins are produced by B-cells/plasma cells The initial reaction is cell mediated immunity in early gingivitis followed by humoral immunity following antigenic insult. The first immunoglobulion produced is secretory that is IgA It later shifts to predominantly humoral immunity with the production of IgG Activation of humoral immunity leads to an accumulation of plasma cells and the production of immunoglobulins. Each gingival plasma cell makes a single class of antibody (ie, IgG, IgA or IgM) specific for a single antigen 133) Teeth that are least affected by periodontal disease are a. Lower first molars and upper anteriors b. Premolars and upper canines c. First molars and upper incisors and premolars d. Lower centrals, laterals and upper molars ANS-(b) Lower Premolrs and Upper canine (Carranzas Clinical Periodontology 10th ed pg no.506-509)

A I P G 2 0 1 1

Teeth least affected by periodontitis is upper canine and lower second premolar Most affected are lower molars.

134) Interdental col is prone to inflammation because a. Epithelium is non keratinized b. Plaque accumulates c. Is difficult to clean d. All of d above Ans (a) Epithelium is non Keratinised (Caranzas Clinical Periodontology 10th page no 48 and Nayak Periodontology-1st ed pg no 221) Repeat AIPG 2009

Inderdental col is non-keratinised Has weakened resistance to inflammatory changes More prone to attack by the injurious agents and less resistant to inflammatory changes Oral hygiene accessibility is not possible. It is the area for food entrapment.

135) Which of the following cells migrate into the sulcus, in largest quantity, as a result of plaque accumulation a. Lymphocytes b. Macrophages c. Neutrophils d. Plasma cells ANS-(c) Neutrophils (Caranzas Clinical Periodontology 10th ed page no 348) Neutrophils are the first cells that migrate in relation to plaque accumulation membrane acts as semipermeable membrane through which

136) Sulcular

DENTAL QUEST Your Quest For The Success In Post Graduate Dental Entrance !

DENTAL

QUEST

AIPG -2011

47

a. Bacterial products pass into saliva b. Fluid from gingiva seeps into the sulcus c. Both a & b d. None of these ANS-(c) Both a & b (Caranzas Clinical Periodontology 10th ed page no 348) Neutrophils are the first cells that migrate in relation to plaque accumulation 137) Which of the following organisms is not implicated in the etiology of periodontal Disease? a. Bacteroides b b. Wolinella c. Neisseria d. Eikenella Ans (c) Neisseria (Caranzas Clinical Periodontology 9th ed pg no 105) Repeat: Kcet -2K, AIIMS Nov 2007 The bacteria associated with periodontal disease are P. Gingivalis, P. Intermedia, A.actinomycetam comitans, Eikenella, fusobacterium Eubacerium species The bacteria associated with periodontal health are (protective species) S.Sanguis S.Mitis Viscosus neisseria Veillonella parvula 138) The bacterial flora associated with Juvenile Periodontitis is mainly a. Gram positive aerobic cocci b. Gram positive anaerobic cocci c. Gram negative aerobic rods d. Gram negative anaerobic rods ANS-(d) Gram negative anaerobe rod (Carranza 9th Ed Pg 107 and 10th Ed Pg 158) Repeat AIPG 2009 A.actinomycetam comitans and Capnocytophaga is the Gram -ve anaerobic rods associated with juvenile periodontitis.

A I P G 2 0 1 1

139) Bacterial enzymes capable of causing the break down of ground substance of
periodontal ligament a. Lysozyme b. Perioxidase c. Hyaluronidase d. Streptokinase ANS (c) hyaluronidase (Carranzas Clinical Periodontology 10th ed pg no 597) Also in National boards Bacterial enzyme capable of altering the ground substance of pdl is hyaluronidase since ground substance is composed of three constituents, hyaluronic acid heparin and chondroitin sulfate.

DENTAL QUEST Your Quest For The Success In Post Graduate Dental Entrance !

DENTAL

QUEST

AIPG -2011

48

140) All of the following can be examined clinically except one. Which of the following
definitely needs Radiographic examination a.Periodontal pocket b.Periodontal abscess c.Anatomic root length d.Furcation involvement Ans (c) Anatomic root length (Caranzas Clinical Periodontology page 8th ed 365,366,354)

CONSERVATIVE AND ENDODONTICS


141) In facial preparation for a porcelain inlay the fragility of porcelain margin requires that a. Cavity walls converge facially b. Cavity walls parallel the facial surface. c. Cavity walls meet the surface of the tooth at right angles d. The axial wall be approximately 2.5mm deeper than normal ANS(c) Cavity walls meet the surface of the tooth at right angles (NBDE PART-II OPERATIVE DENTISTRY APRIL 1979/Q NO-10)
The Type of casvosurface margin recommended for porcelain inlay is a Butt type.

A I P G 2 0 1 1

Cavity preparations for indirect tooth coloured inlays and onlays is essentially same as that of cast metal fabrications minus bevelling and secondary flares. All margins should have 900 cavosurface for marginal strength of the restoration

142)

which of the following should be done for better efficiency and longer shelf life of a carbide bur a. Should rotate slow before entering the cavity b. Should rotate fast before entering the tooth c. Should rotate before entering the mouth d. Should rotate after entering the cavity Ans (b) should rotate fast before entering the tooth Sturdevants Art and Science of Operative Dentistry 4th ed 324 REPEAT AIPG 2009-10 and NBDE Carbide burs do not last long because their brittle blades are easily broken at low speeds. Hence it should be always used at fast speed

143) When pins are used in amalgam,strength of amalgam


a. Always Decreases b. Always Increases c. Increases first followed by a decrease d. May increase or decrease depending on the pin ANS (a) Decreases Frequently repeated question (Sturdevants Art and Science of Operative Dentistry 5th ed page no 810)

The tensile strength and horizontal strength of pin retained amalgam restoration are significantly decreased The pins in amalgam are mainly used for retentive purpose Advantages of pin retained amalgam restorations o Conservative tooth preparation o Can be completed in one appointment o Improved resistance and retention form o In expensive compared to indirect restorations

DENTAL QUEST Your Quest For The Success In Post Graduate Dental Entrance !

DENTAL
o o o 144) a. b. c.

QUEST

AIPG -2011

49

Disadvantages Decreased strength of amalgam Microleakage Dentinal microfractures

Which of the following is correct for a facial wall of cast porcelain inlay The walls should be parallel to the tooth surface Walls should converge facially The walls should diverge facially d. The walls should meet the tooth surface at right angles ANS (c) The walls should diverge facially (Sturdevants Art and Science of Operative Dentistry 4th ed 579-580) All the walls should be diverging for cast inlay whether gold or porcelain in order to seat and remove the restoration and impression without distortion 145) The most common aspect in gold inlay cavity design a. Axiopulpal bevel b. Gingival bevel c. Occlusal bevel d. Axiofacial bevel ANS.(b) Gingival Bevel REF-NBDE PART ll operative dentistry Repeat aipg 2009 The main purpose of a gingival bevel in an inlay or onlay is tat it removes unsupported enamel and compensates for the casting shrinkage In a class II mesio occlusal inlay gingival bevel is at 300 400 and 0.5 to 1mm wide. It means 140-150 degrees cavosurface margin gingival bevel in a class II amalgam is resistance form. The only bevel that is common(of course that is present in amalgam)is gingival bevel in class II restorations inlay 30 45o 30o Amalgam 0 -10 ( ideally zero) 15 20o

A I P G 2 0 1 1

Occlusal bevel Gingival bevel

A reverse bevel is given in inlay for lap sliding fit [MAN-01]. Reverse bevel is placed at the expense of gingival floor in cast restoration.It locks the restoration and prevents proximal displacement. 146) a. b. c. What will you do, if after etching, Class III cavity preparation is contaminated by saliva Wipe the saliva with cotton and continue to restore Wash with water dry area and continue restoration Dry the area and re etch before restoring d. Add excess of composite material and cure it Ans (c) (Sturdevants Art and Science of operative Dentistry 4th ed page no 519) Any contamination by saliva necessitates repeating the placement of etchant for minimum 10 sec following appropriate primer and adhesive placement 147) the a. b. c. A patient reports after 1 hr of restoration of a mandibular molar with a gold inlay with complaint of shooting pain when the teeth come in contact. What must be the cause Supraocclusion Allergic reaction Galvanic current opposing galvanic restoration

DENTAL QUEST Your Quest For The Success In Post Graduate Dental Entrance !

DENTAL

QUEST

AIPG -2011

50

d. Extra cement Ans (c) Galvanic current opposing galvanic restoration (Phillips Science Of Dental Materials 11th ed page no 61,69) Repeat AIPG 2009

Such post operative pain due to galvanic shock occurs immediately after insertion of new restoration and generally occurs when it comes in contact with dissimiliar metals like amalgam. A patient with deflective occlusion complains of pain when taking hot and cold foods has high point. 148) An incipient carious lesion on an interproximal surface is usually located a. At the contact point b. Facial to the contact area c. Lingual to the contact area d. Gingival to the contact area ANS (d) Gingival to the contact area (NBDE PART-II OPERATIVE DENTISTRY APRIL 1978/Q NO-16) Interproximal caries is present gingival to contact area

A I P G 2 0 1 1

ENDODONTICS
149) A young boy had a radio opaque area found at the apex of a permanent tooth with deep
caries which is non-vital. The probable diagnosis is a. Condensing osteitis b. Apical periodontitis c. Chronic periodontitis d. Cementoma ANS (a) condensing osteitis Shafers Textbook of Oral pathology 5th ed page no 686-687 Condensing osteitis More common in mandibular molar area More common in children and young adults Presents large carious lesion Sclerotic bone at periapex 150) a. b.

Intact lamina dura. Reaction to good immunity of the patient No surgery to remove the sclerotic bone

In Cementoma tooth is vital No radio opacity in apical periodontitis Its an endodontic lesion so chronic periodontitis is ruled out

Which is most difficult to diagnose? Necrosed pulp Internal resorption c. Chronic pulpitis d. Acute apical abscess Ans is (c >a ) Necrosed pulp & Chronic Pulpitis (Cohen Pathways of pulp 5th ed page no 272) Both the necrotic pulp and chronic pulpitis may be the answers. Chronic pulpalgia can be differentiated by its clinical symptoms. Necrosis of pulp produces varied clinical signs and symptoms which is difficult to diagnose. But Ingle it is given that the most difficult to diagnose is the chronic pulpitis that is present under a crown restoration Some students have pointed out that this is a direct pick from national boards and the answer is chronic pulpitis.

DENTAL QUEST Your Quest For The Success In Post Graduate Dental Entrance !

DENTAL

QUEST

AIPG -2011

51

151) Tevdek sutures used for endodontic microsurgeries are removed after a. 4872 hours b. 7296 hours c. 7 days d. 10 days Ans (a) 48-72 hours (Practical Clinical Endodontics by Philip Lumley, page no 103 and various journals of endodontics) Repeat AIPG 2009 Tevdek Sutures first braided polyester suture Originally designated for cardiac surgery as they have good strength and low friction while moving through tissue. Recommended for endodontic surgical procedure because of their minimal tissue trauma. 1/3 stronger than silk and are Teflon coated. Non-sticksurface hence less bacteria adhesion and faster healing Allows for early suture removal. Postoperative management: o Sufficient healing should have occurred after 48 hours for suture removal o Should not be left longer than 96 hours due to wicking effect that may cause postoperative infection of the surgical site 152) Recommended p/l ratio of MTA is a. 3:1 b. 1:3 c. 2:3 d. 3:2 ANS (a) 3:1 (Endodontics: Principles and practice by Mahmoud Torabinejad 4th ed Page 32)

A I P G 2 0 1 1

MTA is prepared by mixing the powder with sterile in a 3:1 powder-to-liquid ratio. MTA is currently marketed in 2 forms: gray (GMTA) and white (WMTA). WMTA can be mixed with sterile water, lidocaine, or saline

The mean setting time of MTA is 165 minutes, which is longer than amalgam, Super EBA, and intermediate restorative material (IRM) The pH value of MTA is 10.2 after mixing. This value rises to 12.5 at 3 hours

PEDODONTICS
153) Which surface of primary first molar is most susceptible to dental caries

a. Proximal surface above the contact area b. Proximal surface below the contact area c. Occlusal surface in pits and fissures d. Buccal surface below the height of contour ANS (b) Proximal surface below the contact area (Pediatric dentistry by Pinkham 4th ed pg 206 and Sturdevants Art and science of operative dentistry 5th ed page 120) Interdental caries below the gingival is common in primary teeth where as occlusal caries is common in permanent molar 154) A Radiograph of a first molar shows a very extensive carious lesion that may involve a horn of the dental pulp.The treatment plan of choice is to

DENTAL QUEST Your Quest For The Success In Post Graduate Dental Entrance !

DENTAL

QUEST

AIPG -2011

52

a. Obtain occlusal access to the pulp chamber in anticipation of endo dontic therapy. b. Remove all carious material and cap the obvious pulp exposure c. Remove all carious material,place a sedative dressing and plan for pulpectomy d. Remove a major portion of the decayed tooth and place an interim sedative dressing ANS (d) Remove a major portion of the decayed tooth and place an interim sedative dressing (NBD PART-II OPERATIVE DENTSTRY MARCH 1983 Q NO-22) Place a sedative dressing and plan for a pulpotomy if necessary in next appointment If pulp is not involved indirect pulp capping with Calcium Hydroxide is the indicated treatment. 155) First radiographic sign to diagnose carious involvement in primary tooth a. Radiolucency in furcation area b. Apical PDL radiolucency c. Internal resorption d. Resorption ANS (a) Radiolucency in furcation area (Shobha tondons Textbook of Pedodontics 1st ed page 333) - Frequently repeated national boards question 156) a. A patient reported 20 mins after avulsion of teeth what should be done Rinse with saline, and replant b. Curettage of root/etch c. Perform RCT on tooth d. Remove the PDL and reimplant the tooth ANS (a) Rinse with saline and replant (Grossmanns Endodontic Practice 12th ed page no 381) The teeth should be washed off debris and soil with saline and treated with doxycyline before replanatation

A I P G 2 0 1 1

157) An 8year old child comes to your clinic with large front teeth having jagged margins,
what is the treatment plan for the patient, a. Smoothen the jagged margins and apply fluoride varnish b. Build up other teteth to large size c. Extracting large teeth d. Assure him, and send him back Ans(d) Assure him, and send him back (Whllers dental anatomy 8th pg 105)

Guys this is clearly a case of early duckling stage and mixed dentition stage before prebubertal growth spurt. The central incisors appear larger relative to face size .However the face grows vertically during growth spurt and relatively the large front teeth now will get adjusted to the face proportion. So no need of any treatment. 158) Autism is characterized by a. Incapicitance of motor and mental co ordination b. Mental retardation c. Delayed motor response or lack of motor skills d. Incapicitance of emotional and mental disturbance Ans(a) Incapicitance of motor and mental co-ordination (KAPLAN AND SADDOCK COMPREHENSIVE PSYCHIATRY 8th ed page no 3164) Repeat AIPG 2009 Autism is an incapacitating disturbance of mental and emotional development that causes problems in learning, communicating, and relating to others

DENTAL QUEST Your Quest For The Success In Post Graduate Dental Entrance !

DENTAL

QUEST

AIPG -2011

53

o o

o -

Also called as Kanner syndrome A spectrum of neuropsychiatric disorders Characterized by Deficits in social interaction and communication Unusual and repetitive behavior. Some, but not all, people with autism are non-verbal. Kanners syndrome - described as classical autism, implying low-functioning, Aspergers Syndrome is described as a high-functioning form of autism.

A I P G 2 0 1 1

COMMUNITY DENTISTRY AND MEDICINE


159) Ethnicity refers
a. To self imposed, common origin with similar linguistic and culture characteristics b. To self imposed, common origin with similar linguistic and culture characteristics and Restricted to an area c. To same religion and caste d. Same ancestral origin, cultures and of same geographical area Ans (b) To self imposed, common origin with similar linguistic and culture characteristics and Restricted to an area Ethnicity refers to the culture, customs and traditions acquired from the environment one belongs to. Ethnic groups are marked by regional differences, and not physical characteristics. Ethnic groups have a common history, and may even share their language and religion However, this is not true of every ethnic group. While races may be altered over generations due to crossovers, the ethnicity of a person can easily change if he chooses to adopt the customs and traditions of another ethnic group Race refers to the biological aspects of a person Race does not involve the behavioral or cultural tendencies of a person 160)

a.

India aims to eliminate which of the following diseases by 2015 Malaria b. TB c. Kala azar d. Filariasis

ANS (d) Filariasis (Parks Textbook of Social and Preventive Medicine 20th ed page no 77) India aims to eliminate Filariasis by 2015 according to National Health policy 2002 India alone bears 40% of the global filariasis burden. 161) Which of following regarding Maternal Mortality rate is not true a. Numerator includes total number of female death within 42days of delivery b. Denominator includes still births and abortions c. It is expressed as ratio and rate d. It is expressed per 1000 ANS(b) Denominator includes still births and abortions (Parks Textbook of Social and Preventive Medicine 20th ed page no 479)

Denominator for maternal mortality rate is the total number of live births (and not still births and abortions) Maternal mortality rate=Number of maternal deaths during a given period divided by the

DENTAL QUEST Your Quest For The Success In Post Graduate Dental Entrance !

DENTAL

QUEST

AIPG -2011

54

within

1, 00,000 live births in same time period. It includes only the maternal deaths that arise out of pregnancy, child birth, puerperal causes

42(6 weeks)days of delivery. It does not include pregnancy and incidental causes It is a ratio and and not true rate because denominator does not include the entire population at risk for the outcome described in the numerator (ie. Not all deaths associated with live births) Developing countries still use 1000 for comparison instead of 1 lakh (1,00,000) The target of National health policy is to bring down MMR from 3.1 per 1000 live births to 1 per 1000 by 2010. 162) A randomized trial comparing efficacy of two regimens showed that difference is statically significant with p<0.001 but in reality the two drugs do not differ in their efficacy. This is an example of a. Type-I error b. Type-II error c. I- Alfa d. I-Beta ANS-(a)Type-I error (Soben peters Essentials of Community and Preventive Dentistry 2nd ed pg 43) Type I error is called alpha error Type-II error is called as Beta error First step is Null hypothesis is assumed (i.e. There is no real difference between the means of before and after observation). In general o Null hypothesis is rejected when p value < 0.05 o Null hypothesis is accepted when p value> 0.05 Here the hyposthesis is true that there is no significant difference is seen However Null hypothesis is rejected because p value < 0.05. ( This is seen in above question)

A I P G 2 0 1 1

Type I refers to rejection of null hypothesis in spite being true. Type II refers to acceptance of null hypothesis even if it is not True Incidence is high and prevalance is low means

163)

a. Disease is cured easily or fatal b. Disease is not cured easily c. They are not related d. None of the above ANS (a) Disease is cured easily or fatal (Parks Textbook of Social and Preventive medicine 17th ed page no 53)

In epidemiology the prevalence of a disease in a statistical population is defined as the total number of cases of the disease in the population at a given time, or the total number of cases in the population.

Incidence is a measure of the risk of developing new cases within a specified period of time. Although sometimes loosely expressed simply as the number of new cases during some time period.

DENTAL QUEST Your Quest For The Success In Post Graduate Dental Entrance !

DENTAL

QUEST

AIPG -2011

55

Incidence is high and prevalance is low means the old cases are few in number .This implies either the disease is eliminated or the population affected by the disease is eliminated. 164) a. b. c. d. Negative predictive value 0.75 means.... 75%chances that disease 75%chances that disease 25% chances that disease 25% chances that disease will not be present if test is negative will be present if test is negative will not be present if test is negative will be present if test is negative

ANS (a) 75%chances that disease will not be present if test is negative. (Parks Textbook of Social and Preventive medicine 17th ed page no 112) Negative Predictive Value of Diagnostic Test (NPVDT) is a probability that an individual is not affected with the condition when a negative test result is observed. This measure of accuracy should only be used if the data on the prevalence of condition of interest in given population is available. NPVDT can be determined by calculating: number of true negative results divided by the sum of true negative results plus number of false negative results. Negative predictive value is the proportion of patients with negative test results who are correctly diagnosed. The negative predictive value is the probability a test negative is a true negative For example If a chlamydia test has 80% sensitivity & 80% specificity in a population of 100 with a chlamydia prevalence of 10%: 8 out of 10 true positives test positive 72 out of 90 true negatives test negative Out of 74 negative tests, 82 are true negatives and 2 are false negatives. Therefore, the negative predictive value (NPV) would be 97% (72/74). 97% of people who test negative would actually be negative for chlamydia. 165) Reliability of a screening test refers to? a. Accurately measures what it is supposed to measure b. Gives same values even on repeated testing c. The extent to which the observer can go in finding the result d. Depends on the knowledge of the observer Ans (b) Gives the same values even on repeated testing (Soben peters Essentials ofSocial and Preventive Medicine 2nd ed ed page no 129) Reliability or precision refers to the consistency of a measure. A test is considered reliable if we get the same result repeatedlywhether positive or negative, whether orrect or incorrect. Validity or accuracy is the ability to measure true values. The four measures of validity are sensitivity, speificiity, positive prediction and negative prediction.

A I P G 2 0 1 1

166) Which of the following is most commonly used to measure the variation variables within a group a. Standard error of mean b. Standard deviation c. Coefficient of variation d. Variance Ans (c) Coefficient of Variation

amongst

DENTAL QUEST Your Quest For The Success In Post Graduate Dental Entrance !

DENTAL

QUEST

AIPG -2011

56

(Soban peters Essentials of Preventive and Community Dentistry 3rd ed pg 36 ,37),Repeat AIPG 2010

Dispersion measures include Individual variability Group variability Individual variability: given by range, coefficient of variation, variance and standard deviation Standard deviation [root mean square deviation] is the most important and widely used measure of studying dispersion. It is square root of the mean of the squared deviations from arithmetic mean and is given by formula. Standard deviation is not useful for comparing the variability of two frequency distributions measured in different units or with widely differing means.(so option b not correct) To overcome these problem the relative measure known as coefficient of variation.(So option c is right ) Coefficient of variation is used to measure the variation amongst variables. It is independent of the units of observation. Always expressed in percentage.Higher the value, higher is the variation. o o o

A I P G 2 0 1 1

o
wrong). 167)

Variance is simply the square of of S.D.(so option c is no correct) Group variability between different groups is given by standard error of mean (So option a is

Rural and Urban difference in prevelance is seen in all of the following Except Lung cancer b. Tuberculosis c. Mental Retardation d. Chronic Bronchitis Ans (b) Tuberculosis (Parks Textbook of Social and Preventive medicine 17th ed page no 140)

a.

Guys it is clealy given that there is no difference in prevalence of Tuberculosis and is more or less uniformly distributed in urban,semi urban and rural area 168) Which of the following is correct a. In normal distribution curve mean=median=mode b. Positively skewed distribution curve mean>median c. In Negatively skewed distribution curve mean<median d. All of the above ANS (d) all the above (Soben Peter Essentials of Preventive and community Denistry 2nd ed page no 40) In normal distribution, the curve of distribution of a character is symmetric. It is also known as guassian distribution. Properties of normal curve. o Bell shaped .Total area of this bell shaped curve is one or 100% o Symmetrical o The height of curve is maximum at the mean.Mean is taken as Zero o Mean = Median = Mode o The area under the curve between any two points can be found out in terms of a relation-ship between mean and standard deviation.

DENTAL QUEST Your Quest For The Success In Post Graduate Dental Entrance !

DENTAL

QUEST
covers about 65% of the area covers area. about 95% of

AIPG -2011

57

Mean 1 SD[Standard deviation ] Mean 2 SD Mean _3 SD

Here z score is taken as 2 Here z score is taken as 2.5

covers about 99% of area

A I P G 2 0 1 1

In Positively skewed [right skewed] distribution curve mean > Median In negatively skewed [left skewed] distribution curve Mean < Median

169) Mass Chemoprophylaxis is endemic area is recommended for all of the following except a. Yaws b. Leprosy c. Trachoma d. Filaria ANS (b) Leprosy (Parks Textbook of Social and Preventive medicine 20th ed page no 41,236,287)

o o

Mass chemoprophylaxis is not recommended for the control of leprosy in endemic area Mass chemoprophylaxis /treatment approach is recommended for malaria, yaws,filariasis and trachoma in highly endemic area o For tuberculosis also Mass chemoprophylaxis is not advisable in endemic communities, it can lead to INH resistance and toxicity. BCG vaccination can be given for control of the disease 170) Which of the following is useful in representing the relationship between age and incidence of cancer? a. Diagram b. Line diagram. c. Pie diagram. d. Stem and leaf. ANS (b) line diagram (Parks Textbook of Preventive and Social medicine 19th edition pg no 699 and Soben peters textbook of preventive and community dentistry 2nd ed pg no 30) Different ways to present statistical data Bar chartPresent a set of numbers by the length of bar Types simple, multiple, component Histogram Is a pictorial diagram of frequency distribution. Frequency polygon is obtained by joining midpoint of histogram blocks Line Diagram Show trend in events with passage of time So basically in above question the incidence of cancer with age fits here Pie Charts

DENTAL QUEST Your Quest For The Success In Post Graduate Dental Entrance !

DENTAL

QUEST

AIPG -2011

58

Areas of segments are compared Pictogram

Popular method of presentation of the data to Man in Street Also useful for those who cannot understand orthodox charts The diagrams and the maps is of following types: o One-dimensional diagrams o Line graph, poly graph, bar diagram, histogram, age, sex, pyramid, etc o Two-dimensional diagram o Pie diagram and rectangular diagram o Three-dimensional diagrams o Cube and spherical diagrams. Temperature, rainfall, population growth, birth rates and the death rates. GENERAL MEDICINE 171)
Obesity is not caused by

A I P G 2 0 1 1

a. diabetes mallitus b. estrogen deficiency c. hypothyroidism d. (REF- Davidson's principles and practice of medicine ,20TH ED PG NO 113) Sorry guys..we were not able to find the fourth options.. Anyway.. All the above can cause obesity.. I guess the last must be thr right answer.. Please do look for it in exam.

172) a. b. c.

Cooleys anemia is also known as Vascular hemophilia Christmas disease Thalassemia major d. Thrombocytopenic purpura ANS (c)-Thalassemia (SHAFERS TEXTBOOK OF ORAL PATHOLOGY 5TH ED PG NO 1048)

173) All of the following are used in the treatment of CHF( congestive Heart failure)
treatment except a. Spironolactone b. Nitrates c. Nesiritide d. Trimetazidine ANS (d) Trimetazidine (Harrisons Internal Medicine 17th ed table 227.4)) o Trimetazidine o Calcium channel blocker with cytoprotective action. It acts as an anti oxidant. It is not used in treatment of CHF. o It is used in the treatment of stable angina and myocardial infaraction o Diuretics o Promote the renal excretion of salt and water by blocking tubular reabsorption of sodium and chloride. o Reduces ventricular filling pressures (preload), produces consistent haemodynamic and symptomatic benefits in patients with heart failure o Rapidly improves dyspnoea and peripheral edema o Nesiritide

DENTAL QUEST Your Quest For The Success In Post Graduate Dental Entrance !

DENTAL

QUEST

AIPG -2011

59

Genetically formulated B-type natriuretic peptide which is a venodilator without the adverse effects of the traditional venodilator. o Nitrates o Are Venodilators act by reducing preload and lowering venous pressure, with resulting reduction in pulmonary and dependent oedema 174) CSF glucose level are normal in which type of meningitis a. Viral b. Pyogenic c. Tubercular d. Carcinomatous ANS (a) Viral (Davidsons Principles and Practice of Medicine 20TH ed page no 1225) Repeat AIPG 2003 Condition Normal TB Viral Bacterial Fungal Malignancy Cell type Lymphocytes Lymphocytes Lymphocytes PMNS Lymphocytes Lymphocytes Glucose > 60% of blood glucose [2/3 of blood glucose] Low Norm al Low Low Low Protein Upto 0.4 g/l Elevated Normal Elevated Elevated Elevated

A I P G 2 0 1 1

DENTAL QUEST Your Quest For The Success In Post Graduate Dental Entrance !

DENTAL

QUEST

AIPG -2011

60

175) If pneumothorax occurs in a patient a. Surfactant will be reduced... b. Intrapleural pressure becomes more negative c. Deep breathing is noticed d. Exaggregated breath sounds . ANS.(a) Surfactant will be reduced (Oski's pediatrics: principles & practice by Macmillan 4th ed Page 313) Among given choices , Option A is correct o Decreased surface tension gives rise o pneumothorax. o Pressure in the intra pleural space is greatest. o In pheumothorax, air enters into intra pleural space from outside characterized o Reduced amounts of lung surfactant, cyanosis. o Breathing is shallow and the breath sounds are diminished

A I P G 2 0 1 1

or

normal .

176) The antiviral mechanism of interferon therapy can be best described as


a. Phagocytosis of virus when it enters cell b. Prevention of viral DNA/RNA replication c. Inhibited Protein synthesis of virus d. Prevents Uncoating of virus in Host cell ANS (b) Prevention of viral DNA/RNA replication (Lippincotts Illustrated Reviews Pharmacology 4th ed page no 440)

Although the antiviral effect is not completely understood but it is said to inhibit viral RNA translation and ultimately results its degradation of RNA. o So guys option b is right 177) a. b. Hereditary spherocytosis,all are true except Increased osmotic fragility Increase in surface:volume ratio c. MCHC is increased but MCV is normal. d. They tolerate less osmotic swelling than normal cells Ans (b) Increase in surface: volume ratio (Clinical hematology By Neal 1st ed page no 34) The surface area in Spherocytosis is decreased so the surface area /volume ratio decreases.so the option given in B is wrong. Mean corpuscular haemoglobin concentration (MCHC) means the amount of haemoglobin present in 100ml of RBC .Normal value 33g /100ml of packed cells. In spherocytosis it is around 38/100ml.Option c is right The MCV is normal (80-90 Fl) in spherocytosis. Very rarely in a condition called macrocytic spherocytosis there is temporary swelling and increase in the volume. - Option c is right The osmotic fragility (which reflects the decreased surface-to-volume ratio of red blood cells) is increased they can tolerate less osmotic swelling than normal red blood cells when placed in hypotonic solution option A and D are correct Also note o Heriditary spherocytosis is autosomal dominant disease o RBC is spheroidal, less deformable, and vulnerable to splenic sequestration and destruction. o The defective formation of spectrin and ankyrin which maintains the cell membrane seems to be mainly responsible for spherocytosis. 178) a. b. All of the following agents are used for prophylaxis of migraine except Propanalol Valproate

DENTAL QUEST Your Quest For The Success In Post Graduate Dental Entrance !

DENTAL

QUEST

AIPG -2011

61

c. Topiracamate d. Ethosuximide ANS (d) Ethosuximide (Harrisons Internal medicine 17th ed page no 102) Ethosuximide is an anticonvulsant. Ethosuximide is a nonselective and not very potent blocker of T-type Ca2+ channels. These channels are involved in nociception or pain. Hence ethosuximide is not given in neuropathic pains. Migraine headaches can be treated with two drug approaches:abortive (acute attacks)and preventive (prophylaxis) Prophylaxis

A I P G 2 0 1 1

Beta blockers ----Propranolol, atenolol and metoprolol calcium channel blockers ----- amlodipine, flunarizine and verapamil Anticonvulsants ----sodium valproate, divalproex gabapentin and topiramate Tri-cyclic antidepressents are some of the commonly used drugs. Selective serotonin reuptake inhibitors or serotonin-specific reuptake inhibitor (SSRIs)

DRUGS TO TREAT Acute attacks o Sumatriptan and its congeners,are currently first-line therapy for acute severe migraine attacks in most patients o Ergotamine and sumatriptan useful in aborting migraine can be given orally, nasally, rectally or parenterally. o NSAIDS with caffeine combination o Antiemetics like metoclopromide, domperodone. o Corticosteroidsprednisone is preferred over Dexamethasone o Oedema stagediuretics like furesemide . 179) Bence Jones proteinuria may be seen in a. Alpha heavy chain disease b. Gamma heavy chain disease c. Mu heavy chain disease d. Epsilon heavy chain disease Ans( c) Mu heavy chain disease (Harrisons Internal Medicine 17th ed page no 707)

Bence Jones proteinuria may be seen in Mu heavy chain disease due to excretion of kappa light chain in urine o Mu heavy chain disease B-cell neoplasms and is a variant of Chronic lymphocytic leukaemia (CLL) / small lymphocytic lymphoma (SLL). o There are various rarer conditions that can produce Bence Jones proteins, such as Waldenstrm's macroglobulinemia and other malignances.

180) Gout is Disorder of


a. Purine Metabolism b. Pyrimidine metabolism c. Ketone metabolism d. Protein metabolism ANS.(a) Purine metablosim (Harrisons Internal Medicine 17th ed page no 2444) o o Gout primarily is disorder of purine metabolism Uric acid is final breakdown product of purine degradation in humans

DENTAL QUEST Your Quest For The Success In Post Graduate Dental Entrance !

DENTAL

QUEST

AIPG -2011

62

181) Which of the following method is used for assessment of female fertility during menstrual cycle and best predict time of ovulation a. Basal body temperature b. Fern test c. Spinnbarkeit Phenomenon d. Hormonal study ANS (d) Hormonal Test (GYNAECOLOGICALENDOCRINOLOGY AND INFERTILTY BY SPEROFF 7TH /1036) Of all the options the best option is hormonal study. Predicting and Documenting Ovulation Hormonal Test which measures- (CD=cycle days 3 - 5) (Checks brain - thyroid - adrenal - ovary axis). When grouped together these tests (FSH, LH, E2 (estradiol), Prl (prolactin), DHEA-S, Testosterone, TSH) are called an endocrine profile.They are all normally occurring hormones. 182) Nitrates are not used in the management of a. CCF b. Esophageal spasm c. Renal colic d. Cyanide poisoning ANS.(c) Renal Colic (REF K D Tripathis Essentials of Medical Pharmacology 6th ed page no 524-530) o Nitrates are not commonly used in the management of renal colic though they are capable of relieving spasm but the efficacy is very low 183) All are true about Sitagliptin excepta. Is not given alone, It is given always with sulfonylureas b. It is given in class II diabetics only and not in Type I c. It can be taken with or without food d. It causes some adverse effects ANS (a) Is not given alone, It is given always with sulfonylureas (Diagnosis and Management of Type 2 Diabetes by Steven 1st ed page no 116) Sitagliptin First of a new class of drugs for the treatment of Type 2 diabetes. This group of drugs will be known as DPP-IV (dipeptyl peptidase-IV) inhibitors

A I P G 2 0 1 1

It can be given in initial therapy as an adjunct to exercise and diet or It can be given monotherapy or combination therapy with oral antidiabetics or insulin. It is given in Type II diabetics only. It is not used in Type I diabetics It can be given with or without food. They are incretin enhancers which is a hormone released by gut which inturn reduces the blood glucose level by enhancing the release of insulin

Renal

Well tolerated low rates of adverse effects in clinical trials. Should be given cautiously in failure patients

184) All are true about OPV except


a. Its killed vaccine b. Stored at subzero temperature c. Induces intestinal and humoral immunity both d. Residual neurovirulance is a problem ANS(a) Its a killed Vaccine (Parks Textbook of Social and Preventive Medicine 20 ed pg no 180-182)

DENTAL QUEST Your Quest For The Success In Post Graduate Dental Entrance !

DENTAL

QUEST

AIPG -2011

63

185) a. b. c. d.

Note the word Oral polio vaccine is live attenuated vaccine and is not a killed vaccine Two polio vaccines are used throughout the world to combat poliomyelitis (or polio).

Which is true about multiple myeloma? Presence of IgM hyperviscosity Direct infiltration of kidney by plasma cells IL-B marker for bone lesions Invariably raised alkaline phosphatase Ans-(a) Presence of IgM Hyper Viscosity and (d) Invariably raised alkaline phosphatase (ABC of hemotology by Humes-2nd ed page no 144 and Shafers textbook of Oral pathology 5th ed page no 259-261)

A I P G 2 0 1 1

There will be no direct infiltration of kidneys by plasma cells. Renal failure is due accumulation of proteins, hypercalcemia and primary amyloidosis Multiple myeloma is a tumor of monoclonal plasma cells that accumulate in the bone marrow leading to anemia , bone marrow failure and bone destruction. It arises from the postgerminal centre B-lymphocyte. Most myeloma cells produce and secrete a monoclonal protein, usually a immiunoglobulin.usually paraprotein Ig G protein in 60 percent, Ig A in 25 percent and a light chain in 15% of the cases.( Ig D and Ig M are rare). Light chains are found as Bence Jones proteins in the urine. Hyper production of the M protein leads to hyperviscocity (IgA and IgM due to the size of the immunoglobulin). Hyperviscosity syndrome -includes Multiple myeloma, macroglobulinemia, and polycythemia.

The diagnostic laboratory finding in myeloma is monoclonal hypergammaglobulinemia. IgG myeloma is the most common, followed by IgA myeloma. Other laboratory abnormalities include hyperuricemia (as a result of elevated cell turnover), elevated erythrocyte sedimentation rate (ESR), and increased levels of alkaline phosphatase. Hence, Both are the right options..

186)

The following are true about tetanus acquired through traumatic wound: Clostridium tetani travels via the nerves to the anterior horn cells in the spinal cord The tetanospasmin component of the exotoxin acts on the post-synaptic neurons and prevents impulse transmission. c. The patient should be given antitoxin intravenously d. The presence of Clostridium tetani in the wound can be identified by a positive Nagler Reaction ANS(c) The patient should be given antitoxin intravenously A direct pick from this site http://www.mrcophth.com/MRCOphth/newMCQs/40microbiology.html

a. b.

The diagnosis of tetanus is done by clinical symptoms.

Clostridium tetani bacteria wont spread beyond the wound.It is the toxin that travels( so option A is wrong) The tetanosporin acts on the synapsis to inhibit the inhibitory control of motor nerve impulses so impulse transmission is unimpeded ( so optionB is also wrong)

DENTAL QUEST Your Quest For The Success In Post Graduate Dental Entrance !

DENTAL

QUEST

AIPG -2011

64

The clostridium perfrigens is identified by Nagle reaction and not the Clsotridum tetani( option D is wrong) Treatment includes administration of tetanus immune globulin (TIG), which comprises antibodies that inhibit tetanus toxin (also known as tetanus antitoxins), by binding to and removing unbound tetanus toxin from the body Antitoxin does not reverse the action of toxin that has already caused clinical effects or affect toxin that has ... Clostridium tetani in vitro, and large doses (2 to 4 million units every 6 hours) should be given intravenously. Prevention of tetanus includes vaccination and cleaning the primary wound Prophylaxis Immunization with tetanus immune globulin. Tetanus is not contagious from person to person, and is the only vaccine-preventable disease that is infectious but not contagious. 187) ORF STANDS FOR a. OPEN READING FRAME b. ONCOGENE REMOVING FACTOR c. Oncogenic removing frequency d. Open reduction fracture ANS(a) Open Reading Frame (Harrison's principles of internal medicine Volume 2 17th ed 1937) Repeat AIPG-2010 In molecular genetics, an open reading frame (ORF) is a DNA sequence that does not contain a stop codon in a given reading frame.

A I P G 2 0 1 1

188) Which of the following tests is not used for detection of specific aneuploidy
a. FISH b. RT-PCR c. QF-PCR d. Microarray ANS(d) Microarray (Harrisons Internal Medicine 17th http://www.hkmj.org/article_pdfs/hkm0802p4.pdf)

ed

page

no

406

and

Microarray procedures are the recent aneuploidy tests that gives the Complete copy of entire choromosome sequence instead of detection specific chromosome associated with a particular aneuploidy condition. So option D is wrong 189) a. b. c. d. Dissociated sensory loss in a case of tumor of central spinal cord is due to lesion of Dorsal column fibres Anterior Spinothalamic tract Decussating fibres of lateral spinothalamic tract Cilio spinal centre of spinal cord 8th edition p 140 and

ANS. (c) Decussating fibres of lateral spinothalamic tract (Adams and victor principle of neurology http://en.wikipedia.org/wiki/Dissociated_sensory_loss)

Dissociated sensory loss is a pattern of neurological damage caused by a lesion to a single tract in the spinal cord which involves selective loss of fine touch and proprioception without loss of pain and temperature, or vice-versa.

Loss of pain and temperature are due to damage to the lateral spinothalamic tracts, which cross the central part of the cord close to the level where they enter it and travel up the spinal column on the opposite side to the one they innervate (i.e. they ascend contralaterally).

DENTAL QUEST Your Quest For The Success In Post Graduate Dental Entrance !

DENTAL

QUEST

AIPG -2011

65

Note that a lesion of the lateral spinothalamic tract at a given level will not result in sensory loss for the dermatome of the same level; this is due to the fibers of the tract of Lissauer which transmit the neuron one or two levels above the affected segment (thus bypassing the segmental lesion on the contralateral side).

Loss of fine touch and proprioception are due to damage to the dorsal columns, which do not cross the cord until the brainstem, and so travel up the column on the same side to the one they innervate (i.e. they ascend ipsilaterally).

A I P G 2 0 1 1

This means that a lesion of the dorsal columns will cause loss of touch and proprioception below the lesion and on the same side as it, while a lesion of the spinothalamic tracts will cause loss of pain and temperature below the lesion and on the opposite side to it. Dissociated sensory loss always suggests a focal lesion within the spinal cord or brainstem.

190) All of the following are given during acute attack of migraine except a. Propranolol b. Sumatriptan c. Domperiodon d. Ergotamine . ANS (a) Propranalol (Harrisons Internal Medicine 16th ed page no 93) Propanolol (timolol and metoprolol) are effective for prophylaxis of migraine. Mechanism of action is not known but these drugs are useful for treatment of acute attacks of migraine. Domperidone is a dopamine D2 receptor antagonist with both anti-emetic and gastrokinetic properties.Given intravenously during attacks, the compound relieves the symptoms of nausea and vomiting 191) Tocolytic of choice in heart disease a. Nifedipine b. Atosiban c. Mgso4 d. Salbutamol ANS(b) Atosiban (Cardiac problems in pregnancy: diagnosis and management of maternal and Uriuelkayam-3rd page no 447)

Atosiban (Tractocile, Antocin) is an inhibitor of the hormones oxytocin and vasopressin. It has less effect on hemodynamics and the drug of choice in cardiac patients. Atosiban an oxytocin antagonist seem to be drug of choice at present. Various Tocolytic agent: . Clas II- channel blockers such as nifedipine , Beta-sympathomimetics such as Ritodrine (Yutopar) and Terbutaline Magesium sulphate Postaglandin inhibitorsNSAID such as indomethacin oxytocin and vasopressin inhibitors-- Atosiban 192) Dopamine receptors D2 inhibits Prolactin secretion, so when there is something interfering with this Dopamine receptors then which of the following will not occur. a. Visual field defects

DENTAL QUEST Your Quest For The Success In Post Graduate Dental Entrance !

DENTAL
b. c. d.

QUEST

AIPG -2011

66

Gonadal dysfunction Headache Exccessive lactation

Ans (a) Visual field defect (Yen and Jaffe's reproductive endocrinology: physiology, pathophysiology6th/456 GuysThis is a logic question and seems all the options are correct but visual disturbances will not occur in all the cases. They occur only in pituitary adenomas which compress the optic chaisma 193) The disorder characterized by craniosynostosis, craniofacial anomalies, severe , Symmetrical syndactyly (cutaneous and bony fusion) of the hands and feets along with preaxial polysyndactyly and variable soft tissue syndactyly is a. Carpenter Syndrome b. Crouzon Syndrome c. Apert Syndrome d. Downs Syndrome Ans (a) Carpenter Syndrome (Nelson Textbook of Pediatrics -16th ed page no 1813) Repeat AIPG 2009 Carpenter Syndrome Autosomal recessive inheritance. Craniosynostosis is associated with mental retardation and preaxial polysyndactyly of the feet. Soft tissue polydactyly of hands is also present. Patella is displaced Children have a short stature, are obese and often suffer from congenital heart disease 194) Haemopoietic stem cells differ from committed progenitor cells a. Long term reconstitution of bone marrow b. High differenciating capacity than proginetor cell c. Self renewal d. All of the above Ans (a) Long term reconstitution of bone marrow (Clinical Bone Marrow and Blood Stem Cells transplantation by Kerry Atkinson 1st ed page no 38)

A I P G 2 0 1 1

"Primitive" stem cells are more likely to be quiescent (ie, stem cells capable of long-term hematopoietic reconstitution) while more mature stem cells may be in cell cycle

GENERAL SURGERY
195) Minimum GCS score is a. 9 b. 1 c. 3 d. 0 ANS (c) 3 (Bailey and Love Short Practice of Surgery - 24th ed pg 601) Glasgow coma scale Maximum score in each category indicates best response, i.e., 4+5+6=15 Also the minimum score is 3. maximum GCS score 15 and minimum 3 Eye Verbal Motor opening response activity

DENTAL QUEST Your Quest For The Success In Post Graduate Dental Entrance !

DENTAL

QUEST
5:oriented and talks 4:disoriented and talks 3:inappropriate words 2:incomprehensible sounds 1:decerebrate

AIPG -2011
6:verbal command 5:localizes pain 4:withdraws to pain 3:decorticate 1:no response

67

4:spontaneous 3:verbal stimuli 2:painful stimuli 1:no response

196) Which of the following is not given parenteral nutrition a. Fibre b. Micronutrient c. Carbohydrate d. SFat ANS(a) Fibre (Schwartz Principle of surgery 8th edn pg no 36) Fibres are not included in parenteral nutrition and Parenteral nutrition involves the continuous infusion of hyperosmolar solution containing carbohydrates, proteins, fat and other nutrient. 197) Paradoxical movement of chest in a patient who has suffered trauma is called a. Flial chest b. Cardiac temponade c. Dyspnoea d. Pneumothorax ANS (a) Flial chest (BAILEY AND LOVE Short Practise of Surgery 24th ed page no 869) AIPG 2009

A I P G 2 0 1 1

Flail chest is a situation in which a portion of the rib cage is separated from the rest of the chest wall, usually due to a severe blunt trauma, such as a serious fall or a car accident. Flail chest is a serious condition that can lead to long-term disability and even death. For a person to draw a breath, the muscles around the rib cage and the diaphragm have to move to expand the chest cavity. There is a Creation of vacume that is filled as air enters the lungs. If this expansion is hindered, the ability to draw air into the lungs is diminished. A flail chest is a chest in which sections of broken ribs are isolated from, and interfering with, normal chest movements. That means the chest cannot expand properly and cannot properly draw air into the lungs. This is why stabilization after blunt trauma is important. 198) a. All are used in pitutary adenoma except Orlistat b. Bromocriptine c. Octreotide. d. Letrozol ANS (a) Orlistat (Harrison Internal Medicine 17th ed page no 1513, 2257,2319) Orlistat Also known as tetrahydrolipstatin, Used to treat obesity. Preventing the absorption of fats from the human diet, Treatment options of pituatory adenoma Prolactinomas decrease tumor size as well as alleviates symptoms, Octreotide for Somatotrophic adenomas Letrozole - third-generation nonsteroidal aromatase Efforts have been made to use a progesterone antagonist for the treatment of prolactinomas, but so far have not proved successful.

DENTAL QUEST Your Quest For The Success In Post Graduate Dental Entrance !

DENTAL

QUEST

AIPG -2011

68

Surgery is a common treatment specilly.Trans-sphenoidal adenectomy is preferred

199) a. b. c. d.

An Ann- arbour classification Stage III corresponds to : Involvement of lymph node regions or lymphoid structures on both sides of the diaphragm. Involvement of two or more lymph node regions on the same side of the diaphragm Involvement of one lymph node or a single region Diffuse involvement of extralymphatic lymph nodes.

Ans-(a) Involvement of lymph node regions or lymphoid structures on both sides of the diaphragm. (Shafers Textbook of oral pathology 5th Ed pg No 256) Ann Arbor staging is the staging system for lymphomas, both in Hodgkin's lymphoma (previously called Hodgkin's disease) and Non-Hodgkin lymphoma (abbreviated NHL). It was initially developed for Hodgkin's, but has some use in NHL. It has roughly the same function as TNM staging in solid tumors. The principal stage is determined by location of the tumor: Stage I indicates that the cancer is located in a single region, usually one lymph node and the surrounding area. Stage I often will not have outward symptoms. Stage II indicates that the cancer is located in two separate regions, an affected lymph node or organ and a second affected area, and that both affected areas are confined to one side of the diaphragm - that is, both are above the diaphragm, or both are below the diaphragm. Stage III indicates that the cancer has spread to both sides of the diaphragm, including one organ or area near the lymph nodes or the spleen. Stage IV indicates diffuse or disseminated involvement of one or more extralymphatic organs, including any involvement of the liver, bone marrow, or nodular involvement of the lungs

A I P G 2 0 1 1

200) a. b. c. d.

Number of cases at particular age in a given place is called/given by (CPAP values) Incidence Proportion Prevalence Test of variance Ans( a) Incidence (Soben Peters Essentials of Preventive and Community Dentistry 2nd ed page no 88 89)

DENTAL QUEST Your Quest For The Success In Post Graduate Dental Entrance !

S-ar putea să vă placă și